SlideShare a Scribd company logo
1 of 62
Download to read offline
StuDocu is not sponsored or endorsed by any college or university
QUIZ, questions and answers
Bachelor of Science Nursing (La Consolacion University Philippines)
StuDocu is not sponsored or endorsed by any college or university
QUIZ, questions and answers
Bachelor of Science Nursing (La Consolacion University Philippines)
Downloaded by John Paul Magbitang (johnpaulmagbitang04@gmail.com)
lOMoARcPSD|15752181
TAXATION QUIZZER PART 1
BASIC PRINCIPLES OF TAXATION
1. Which theory in taxation states that without taxes, a government
would be paralyzed for lack of power to activate and operate it,
resulting in its destruction?
a. Power to destroy theory
b. Lifeblood theory
c. Sumptuary theory
d. Symbiotic doctrine
2. The actual effort exerted by the government to effect the exaction
of what is due from the taxpayer is known as
a. Assessment.
b. Levy.
c. Payment.
d. Collection.
3. Although the power of taxation is basically legislative in
character, it is NOT the function of Congress to
a. Fix with certainty the amount of taxes.
b. Collect the tax levied under the law.
c. Identify who should collect the tax.
d. Determine who should be subject to the tax.
4. An example of a tax where the concept of progressivity finds
application is the
a. Income tax on individuals.
b. Excise tax on petroleum products.
c. Value-added tax on certain articles.
d. Amusement tax on boxing exhibitions.
5. Ligaya Educational Foundation, Inc., a stock educational
institution organized for profit, decided to lease for commercial
use a 1,500 sq. m. portion of its school. The school actually,
directly, and exclusively use the rents for the maintenance of its
school buildings, including payment of janitorial services. Is the
leased portion subject to real property tax?
a. Yes, since Lualhati is a stock and for profit educational
institution.
b. No, since the school actually, directly, and exclusively
used the rents for educational purposes.
c. No, but it may be subject to income taxation on the rents it
receives.
d. Yes, since the leased portion is not actually, directly, and
exclusively used for educational purposes.
6. Which among the following concepts of taxation is the basis for
the situs of income taxation?
a. Lifeblood doctrine of taxation
b. Symbiotic relation in taxation
Downloaded by John Paul Magbitang (johnpaulmagbitang04@gmail.com)
lOMoARcPSD|15752181
c. Compensatory purpose of taxation
d. Sumptuary purpose of taxation
DONOR’S TAX
7. Which of the following transactions is deemed a taxable gift?
a. Condonation or remission of a debt
b. Sale of residential house and lot for less than adequate and
full consideration in money or money’s worth
c. Both (a) and (b)
d. Neither (a) nor (b)
8. Which of the following statements relative to donor’s tax is
false?
a. The spouses shall file separate donor’s tax returns where
the thing donated is common property.
b. Each parent shall be entitled to the P10,000 exemption on
account of marriage of a child.
c. Exemptions and deductions cannot be claimed where the 30%
tax rate on stranger is applicable.
d. None of the foregoing.
9. The spouses Esme and Carlisle wanted to donate a parcel of land to
their son Edward who is getting married in December, 2016. The
parcel of land has a zonal valuation of P420,000.00. What is the
most efficient mode of donating the property?
a. The spouses should first donate in 2016 a portion of the
property valued at P20,000, then spread the P400,000 equally
for 2017, 2018, 2019 and 2020.
b. Spread the donation over a period of 5 years by the spouses
donating P100,000 each year from 2016 to 2020.
c. The spouses should each donate a P110,000 portion of the
value of the property in 2016 then each should donate
P100,000 in 2017.
d. The spouses should each donate a P100,000 portion of the
value of the property in 2016, and another P100,000 each in
2017. Then, in 2018, Esme should donate the remaining
P20,000.
10. Exempted from donor’s taxation are gifts made
a. For the use of the barangay.
b. In consideration of marriage.
c. To a school which is a stock corporation.
d. To a for-profit government corporation.
11. Caroline donated P110,000.00 to her friend Vicky who was getting
married. Caroline gave no other gift during the calendar year.
What is the donor's tax implication on Caroline’s donation?
a. The P100,000 portion of the donation is exempt since given
in consideration of marriage.
b. A P10,000 portion of the donation is exempt being a donation
in consideration of marriage.
Downloaded by John Paul Magbitang (johnpaulmagbitang04@gmail.com)
lOMoARcPSD|15752181
c. Caroline shall pay a 30% donor's tax on the P110,000
donation.
d. The P100,000.00 portion of the donation is exempt under the
rate schedule for donor's tax.
12. A non-stock, non-profit school always had cash flow problems,
resulting in failure to recruit well-trained administrative
personnel to effectively manage the school. In 2017, Don Leon
donated P100 million pesos to the school, provided the money shall
be used solely for paying the salaries, wages, and benefits of
administrative personnel. The donation represents less than 10% of
Don Leon's taxable income for the year. Is he subject to donor's
taxes?
a. No, since the donation is actually, directly, and
exclusively used for educational purposes.
b. Yes, because the donation is to be wholly used for
administration purposes.
c. Yes, since he did not obtain the requisite NGO certification
before he made the donation.
d. No, because the donation does not exceed 10% of his taxable
income for 2017.
13. What law shall govern the imposition of donor’s tax?
a. The law in force at the time of perfection of the donation
b. The law in force at the time of completion of the donation
c. The law in force at the time of perfection or completion
depending upon the agreement of the parties
d. None of the choices
14. Andy, married, donated a land commonly owned by him and her spouse
worth P500,000 to her friend Joan. Only Andy signed the deed of
donation. Joan assumed P200,000 unpaid mortgage on the property.
How much is the donor’s tax due?
a. P6,000 c. P1,000
b. P90,000 d. P45,000
500k - 200k = 300k x 30% = 90,000
15. Dondie, resident citizen, made the following donations on April 28
of the current year:
 To his sister, Donna,P175,000 worth of property situated in
Paris, France. The donor’s tax paid is P40,000.
 To Dara, his girlfriend in the Philippines, jewelry valued at
P225,000.
 To International Rice Research Institute, cash amounting to
P50,000.
The donor’s tax due after tax credit is –
a. P69,000 c. P29,000
b. P38,813 d. P5,625
Downloaded by John Paul Magbitang (johnpaulmagbitang04@gmail.com)
lOMoARcPSD|15752181
225,000 x 30% = 67,500 + (175,000 - 100,000 x 2% Tabular) = 69,000 -
(69,000 x 175k/400k) = 38,813
ESTATE TAX
16. Which of the following is not a part of the gross estate?
a. Conjugal property
b. Community property
c. Exclusive property of the decedent
d. Exclusive property of the surviving spouse
17. Who among the following transferors is not liable for estate tax
on the property transferred during his lifetime?
a. The testator who bequeaths property to his heirs in a last
will and testament executed and probated during his lifetime
b. The donor who reserves his right to amend or revoke the
donation of property in favor of the donee
c. The donee of an appointed property who is required under a
power of appointment to transfer such property upon death to
his eldest child
d. The transferor of personal property who sold it for
insufficient consideration
18. Which of the following properties of the spouses will be part of
common properties under a regime of Conjugal Partnership of Gains?
a. Land inherited during the marriage
b. Fruits of land inherited
c. Jewelry inherited during the marriage
d. Building donated before marriage
19. Which of the following items is not considered as a “special
deduction” in computing the taxable net estate of the decedent?
a. Vanishing deduction
b. Medical expenses
c. Standard deduction
d. Family home allowance
20. When the payment of estate tax will cause undue hardship upon the
heirs or the estate which is undergoing judicial settlement before
the court, the BIR Commissioner may grant an extension for a
period not exceeding:
a. 5 years
b. 3 years
c. 2 years
d. 1 year
Next three (3) questions are based on the following:
Orland, married resident citizen, died on August 20, 2016. The estate
reported the following assets and deductions:
Conjugal Properties:
Fishpond, Bulacan P1,500,000
Family Home, Makati 1,500,000
Cash in bank 900,000
Downloaded by John Paul Magbitang (johnpaulmagbitang04@gmail.com)
lOMoARcPSD|15752181
Exclusive Properties of Orland:
Land, inherited from his father who
died on July 20, 2012. The value of
land at the time of inheritance was
P210,000. The land was mortgaged for
P30,000 which was unpaid at the time of
death of his father,P10,000 of which
was paid by Orland before he died.
P400,000
Land, donated on February 14, 2013 by
his mother who died on November 2,
2014. The value of the land when
donated was P500,000 while upon death
of his mother was P400,000.
600,000
Exclusive Properties of Wife:
Farm in Laguna, acquired before
marriage
2,000,000
Deductions claimed:
Funeral expenses 250,000
Fire loss of apartment (occurred 4
months after death)
80,000
Bad debts (represents unpaid receivable
from Bert, an insolvent)
100,000
Mortgage on inherited land 30,000
Vanishing deduction on inherited
land
40,000
Vanishing deduction on donated land 20,000
Standard deduction 2,000,000
21. The gross estate is:
a. P4,000,000 c. P7,000,000
b. P4,710,000 d. P5,000,000
22. The vanishing deduction is:
a. P184,000 c. P255,760
b. P220,800 d. P292,560
23. The net taxable estate is:
a. P819,200 c. P829,200
b. P804,200 d. P579,200
Downloaded by John Paul Magbitang (johnpaulmagbitang04@gmail.com)
lOMoARcPSD|15752181
SOLUTION:
Properties Exclusive Conjugal Total
Fishpond, Bulacan P1,500,000 1,500,000
Family Home, Makati 1,500,000 1,500,000
Cash in bank 900,000 900,000
Land, inherited from Father
died on July 20, 2012. 400,000 400,000
Land, Donated from Mother
on Feb 14, 2013. 600,000 600,000
Claims to Insolvent 100,000 100,000
GROSS ESTATE 1,000,000 4,000,000 5,000,000
Allowable Deductions:
Funeral Expenses (200,000) (200,000)
Fire Loss (80,000) (80,000)
Bad Debts (100,000) (100,000)
Mortgage on Land (20,000) (20,000)
Vanishing Deductions* (220,800) (220,800)
Net Estate before Special
Deductions 759,200 3,620,000 4,379,200
Special Deductions:
Family Home (1,500,000 x 1/2) (750,000)
Special Deduction (1,000,000)
Share of Surviving Spouse (1,810,000)
NET TAXABLE ESTATE 819,200
*VANISHING DEDUCTIONS:
Land Inherited by Father:
Value at the time of Death 210,000
Less: Mortgage Paid (10,000)
Initial Basis 200,000
Pro rata: 200/5000 x 400,000 (16,000)
Final Basis 184,000
Vanishing Rate (4 years but not more 5 yrs) 20%
Vanishing Deduction 36,800
Land Donated by Mother:
Value at the time of Donation 500,000
Initial Basis 500,000
Pro rata: 500/5000 x 400,000 (40,000)
Final Basis 460,000
Vanishing Rate (3 years but not more 4 yrs) x 40%
Vanishing Deduction 184,000
TOTAL VANISHING DEDUCTIONS 220,800
**If TRAIN Law is applied the Estate tax is P 49,152 (P819,200 x 6%)
Downloaded by John Paul Magbitang (johnpaulmagbitang04@gmail.com)
lOMoARcPSD|15752181
24. Abe, married resident alien, died on January 15, 2017. She left
the following properties, expenses and obligations:
Community properties, Philippines (including
family home valued at P1,800,000)
P5,000,000
Community properties, Abroad 2,000,000
Exclusive properties, Philippines 3,000,000
Actual funeral expenses 300,000
Judicial expenses 200,000
Medical expenses (incurred w/in 1yr. before death) 600,000
Devise to National Gov’t 50,000
Legacy to Local Gov’t 70,000
The net taxable estate is:
a. P3,780,000 c. P3,580,000
b. P3,680,000 d. P3,530,000
RESIDENT ALIEN
Particulars Exclusive Community Total
All Properties w/i & w/o 3,000,000 7,000,000 10,000,000
Funeral Expense (200,000) (200,000)
Judicial Expense (200,000) (200,000)
Transfers (50,000 + 70,000) (120,000) (120,000)
Gross Estate 2,880,000 6,600,000 9,480,000
Share of Surviving Spouse (3,300,000)
Medical Expenses (500,000)
Family Home (1/2 of 1,800,000) (900,000)
Standard Deductions (1,000,000)
Net Estate 3,780,000
25. Based on the above problem, if the decedent is a non-resident
alien, how much is the net taxable estate?
a. P2,755,000 c. P5,220,000
b. P2,880,000 d. P5,380,000
NON - RESIDENT ALIEN
Particulars Exclusive Community Total
All Properties w/i Only 3,000,000 5,000,000 8,000,000
Funeral Expense 200,000
Judicial Expense 200,000
Allowable Deduction 400,000 x 8M/10M (320,000) (320,000)
Transfers(50,000 + 70,000) (120,000) (120,000)
Gross Estate 2,880,000 4,680,000 7,560,000
Share of Surviving Spouse (2,340,000)
Net Estate 5,220,000
26. Arthur, Filipino, married died leaving the following estate:
Car acquired before marriage by Arthur P 300,000
Downloaded by John Paul Magbitang (johnpaulmagbitang04@gmail.com)
lOMoARcPSD|15752181
Car acquired before marriage by wife 450,000
House and lot acquired during marriage 1,500,000
Jewelries of wife 100,000
Personal properties inherited by Arthur during
marriage
250,000
Benefits from SSS 50,000
Retirement benefits 150,000
Proceeds of group insurance taken by his employer 75,000
Land inherited by the wife during marriage 1,000,000
Income earned from the land inherited by wife (25%
of which was earned after death)
200,000
16. How much is the gross estate if the property relationship is
conjugal partnership of gains is:
a. P2,600,000 c. P1,950,000
b. P3,600,000 d. P2,200,000
Car acquired before marriage by Arthur P 300,000
House and lot acquired during marriage 1,500,000
Personal properties inherited by Arthur during marriage 250,000
Income earned from the land inherited by wife
(25% of which was earned after death) 150,000
Gross Estate 2,200,000
27. Based on the preceding number, the gross estate if the property
relationship is absolute community of property is:
a. P2,600,000 c. P1,950,000
b. P3,600,000 d. P2,500,000
Car acquired before marriage by Arthur P 300,000
Car acquired before marriage by wife 450,000
House and lot acquired during marriage 1,500,000
Jewelries of wife 100,000
Personal properties inherited by Arthur during marriage 250,000
Gross Estate 2,600,000
VAT
28. LBJ made the following sales during the 12-month period:
Sales, VAT taxable transactions P1,500,000
Sales, VAT zero-rated transactions 400,000
Sales, VAT exempt transactions 100,000
Total P2,000,000
Which of the following statements is correct?
a. LBJ may not register under the VAT system because his sales from
VAT taxable transactions did not exceed P1,919,500.
b. LBJ may not register under the VAT system because his sales from
VAT taxable and zero-rated transactions did not exceed
P1,919,500.
Downloaded by John Paul Magbitang (johnpaulmagbitang04@gmail.com)
lOMoARcPSD|15752181
c. LBJ is required to register because his total 12-month sales
exceeded P1,919,500.
d. None of the foregoing.
29. Which of the following is exempt from VAT?
a. Common carriers transporting passengers by air within the
Philippines
b. Common carriers transporting passengers by sea within the
Philippines
c. Common carriers transporting passengers by land within the
Philippines
d. Common carriers transporting cargoes by air within the
Philippines
30. Which statement is correct about value-added tax on goods or
properties sold?
a. It is based on gross sales and not on net sales;
b. May be due even if the goods or properties were not actually
sold;
c. Does not cover goods exported;
d. It forms part of the selling expense of the trader.
31. For value-added tax purposes, which of the following transactions
of a VAT-registered taxpayer may not be zero-rated?
a. Export sales
b. Foreign currency denominated sales
c. Sale of goods to the Asian Development Bank
d. Sale of goods to an export oriented enterprise
32. A subdivision developer sold five (5) residential house and lots,
each to different vendees, for P3,000,000 per lot, or a total
sales of P15,000,000 for the taxable period.
These sales shall be classified as:
a. 12% VAT transactions
b. 0% VAT transactions
c. VAT exempt transactions
d. None of the foregoing
**3,199,000 each is the threshold
33. CP operated a retail business that had been generating sales not
exceeding the threshold for VAT exempt persons. However, he
desires to be registered under the VAT system for the first time
in order to benefit from input tax credits.
What benefit may CP be entitled to once he registers under the VAT
system?
a. Tax refund
b. Presumptive input tax credit
c. Transitional input tax credit
d. None of the foregoing
Downloaded by John Paul Magbitang (johnpaulmagbitang04@gmail.com)
lOMoARcPSD|15752181
34. What institution is required to deduct and withhold a final VAT of
5% on the purchase of goods or services subject to VAT?
a. National government or any political subdivision thereof
b. Government-owned or controlled corporations
c. Both (a) and (b)
d. Neither (a) nor (b)
35. In the value-added tax on sale of services, the output tax is
computed:
a. On the billings of the month
b. On collections of the month on all billings made
c. On the contract price of contracts completed during the
taxable period
d. Only and strictly on labor performed under the contract for
services
36. Which statement is wrong?
a. There is a transitional input tax from purchases of goods or
properties;
b. There is a transitional input tax from purchases of
services;
c. There is a transitional input tax from purchases of
materials;
d. There is a transitional input tax from purchases of
supplies.
37. Which of the following statements is correct on the inventory
balance in the financial statements at any given date of a VAT-
registered person?
a. Balance, net of input taxes
b. Balance, inclusive of input taxes
c. Balance on which the transitional input tax is computed
annually
d. Balance where the VAT thereon may be calculated by
multiplying it by 12%
38. Genson Distribution Inc., a VAT taxpayer, had the following data
in a month:
Cash sales P200,000
Open account sales 500,000
Consignment:
0 to 30 days old (on which there
were remittances from consignees of
P200,000) 600,000
31 to 60 days old 700,000
61 days old and above 900,000
How much is the output tax?
a. P348,000 c. P264,000
b. P216,000 d. P108,000
Downloaded by John Paul Magbitang (johnpaulmagbitang04@gmail.com)
lOMoARcPSD|15752181
Cash sales P200,000
Open account sales 500,000
Consignment:
0 to 30 days old (on which there were
remittances from consignees of P200,000) 200,000
61 days old and above 900,000
Total VATABLE SALES 1,800,000
VAT RATE 12%
OUTPUT VAT 216,000
42. The financial records of Benz Corp., a VAT-registered taxpayer,
for the taxable year 2016 disclosed the following:
Local sales to private entities 1,500,000
Export Sales 500,000
Local sales to government 800,000
How much is the total sales subject to value-added tax?
a. P2,800,000 c. P2,000,000
b. P2,300,000 d. P1,500,000
Local sales to private entities 1,500,000
Export Sales 500,000
Local sales to government 800,000
Total VATABLE SALES 2,800,000
43. Mantika Corp., a VAT-registered Corp., is a producer of cooking
oil from coconut and corn. It had the following data for the month
of January 2017:
Sales, gross of VAT P 784,000
Corn & Coconut, 12-31-16 50,000
Purchases of Corn & Coconut 330,000
Corn & Coconut, 1-31-17 20,000
Purchases from VAT suppliers, VAT included:
Packaging Materials 56,000
Supplies 16,800
The value-added tax payable for the month:
a. P56,060 c. P60,650
b. P54,900 d. P63,000
Sales, gross of VAT P 784,000
Output TAX 84,000
Purchases of Corn & Coconut (330,000 x4%) (13,200)
Purchases from VAT suppliers, VAT included:
Packaging Materials 56,000
Supplies 16,800 72,800 x3/28 (7,800)
VAT PAYABLE 63,000
Downloaded by John Paul Magbitang (johnpaulmagbitang04@gmail.com)
lOMoARcPSD|15752181
44. Bunga Inc., a VAT taxpayer, is engage in the business of
processing of fruits. Its data on sales and purchases for the
month of August are provided below:
Sales P200,000
Purchases:
Fresh Fruits 30,000
Raw sugarcane 12,000
Tin Can, gross of VAT 12,320
Paper Labels, net of VAT 5,000
Cardboard for boxes, net of VAT 8,000
Freight, gross of VAT (50% still unpaid) 10,080
How much is the value-added tax payable?
a. P20,580 c. P19,380
b. P18,900 d. P20,100
Sales P200,000 Output Tax 24,000
Purchases:
Fresh Fruits 30,000
Raw sugarcane 12,000
Tin Can, gross of VAT 12,320 Input Tax (1,320)
Paper Labels, net of VAT 5,000 Input Tax ( 600)
Cardboard for boxes, net of VAT 8,000 Input Tax ( 960)
Freight, gross of VAT
(50% still unpaid) 10,080 Input Tax ( 540)
VAT PAYABLE 20,580
45. Bahay Kubo Inc. is a real estate dealer. Details of its sales
during the year showed the following:
Date of sale June 2, 2017
Consideration in the deed of sale P 5,000,000
Fair market value in the assessment rolls 4,800,000
Zonal Value 5,200,000
Schedule of payments:
June 2, 2017 1,000,000
June 2, 2018 2,000,000
June 2, 2019 2,000,000
How much is the output tax to be recognized for the June 2, 2018
payment?
a. P0 c. P249,600
b. P124,800 d. P624,000
**Zonal Value 5,200,000 x 12% = 624,000 x 2M/5M =
249,600 Output Tax for 2018
46. Assuming that the scheduled payment on June 2, 2017 is P2,000,000,
how much is the output tax to be recognized for the June 2, 2019
payment?
a. P0 c. P249,600
b. P124,800 d. P624,000
Downloaded by John Paul Magbitang (johnpaulmagbitang04@gmail.com)
lOMoARcPSD|15752181
Zero as in 0 for the sale will no longer qualify as Installment
Sales.
47. Mr. Karpentero, a vat-registered building contractor, has the
following data on gross receipts in a month, any tax not included:
 From Mr. A, a private property owner, final payment on the
contract price, net of 5% agreed retention fee of P2,850,000
 From Mr. B, a payment of 5% retention on the contract price
previously made by him P100,000
 From Mr. C, for materials used in the construction 500,000
How much is the output tax?
a. P414,000 c. P72,000
b. P342,000 d. P62,000
Final Payment on Contracts 2,850,000
Retention 100,000
Materials 500,000
TOTAL Receipts 3,450,000
VAT Rate 12%
Output VAT 414,000
48. COC Inc., in its first month of operation, and as a VAT taxpayer,
purchased various fixed assets. Purchases of fixed assets in the
first month were as follows:
Light equipment, with a useful life of 3 years P 300,000
Heavy equipment, with a useful life of 10 years 4,000,000
How much is the input tax available for the month?
a. P516,000 c. P480,000
b. P9,000 d. P8,600
Light Equipment 300,000/36 x 12% 1,000
Heavy Equipment 4,000,000/60 x 12% 8,000
Total Input VAT 9,000
49. Kusina Co., had its kitchen assembled by a VAT taxpayer. It took
six months for the contractor to finish the work. Kusina Co.
purchased materials in July from VAT suppliers at a cost of
P500,000, VAT not included. Payment to the contractor in July 2017
on the Construction in Progress, VAT not included was:
On contractor’s billing in June P100,000
On contractor’s billing in July 70,000
Downloaded by John Paul Magbitang (johnpaulmagbitang04@gmail.com)
lOMoARcPSD|15752181
The input tax available in July is:
a. P0 c. P60,000
b. P80,400 d. P20,400
Materials from VAT Supplier 500,000
On contractor’s billing in June 100,000
On contractor’s billing in July 70,000
Total 670,000
VAT Rate 12%
Output Tax 80,400
50. Data from the books of accounts of a VAT taxpayer for February:
Domestic Exports
Sales P 2,000,000 8,000,000
Purchases:
From VAT Suppliers:
Goods for sale 600,000 2,400,000
Supplies & services 90,000 360,000
From Suppliers Paying percentage tax:
Goods for sale 100,000 1,500,000
Supplies & services 20,000 80,000
If the input taxes attributable to zero-rated sales are claimed as
tax credit, the net value-added tax refundable is:
a. P136,000 c. P145,000
b. P203,924.70 d. P174,000
Output Tax (2,000,000 x 12%) 240,000
Input Tax on Domestic Sales (690,000 x 12%) (82,800)
Input Tax on Zero Rated (2,760,000 x 12%) (331,200)
Refundable VAT (174,000)
INCOME TAXATION
51. C. Lee, Chinese national, arrived in the Philippines on January 1,
2012 to visit his Filipina paramour. He planned to stay in the
country until December 31, 2016, by which time he would go back to
his legal wife and family in China. C. Lee derived income during
his stay here in the Philippines.
For the taxable year 2012, C. Lee shall be classified as a:
a. Resident alien
b. Non-resident alien engaged in trade or business in the
Philippines
c. Non-resident alien not engaged in trade or business in the
Philippines
d. Special alien employee
52. The following individual taxpayers are subject to the graduated
income tax rates of 5%-32%, except
a. Filipino citizens
b. Resident aliens
c. Non-resident alien engaged in trade or business in the
Philippines
Downloaded by John Paul Magbitang (johnpaulmagbitang04@gmail.com)
lOMoARcPSD|15752181
d. Non-resident alien not engaged in trade or business in the
Philippines
53. In which of the following cases will the dividend income from a
foreign corporation be classified as “income without”
a. Less than 50% of the foreign company’s gross income for the
preceding three (3) years prior to the dividend declaration
was derived from sources within the Philippines.
b. 50% of the foreign company’s gross income for the preceding
three (3) years prior to the dividend declaration was
derived from sources within the Philippines.
c. More than 50% of the foreign company’s gross income for the
preceding three (3) years prior to the dividend declaration
was derived from sources within the Philippines.
d. Always classified as income without”.
54. D’ Lion, Inc., a Philippine corporation, sold through the local
stock exchange 10,000 PLDT shares that it bought 2 years ago. D’
Lion sold the shares for P2 million and realized a net gain of
P200,000. How shall it pay tax on the transaction?
a. It shall declare a P2 million gross income in its income tax
return, deducting its cost of acquisition as an expense.
b. It shall report the P200,000 in its corporate income tax
return adjusted by the holding period.
c. It shall pay 5% tax on the first P100,000 of the P200,000
and 10% tax on the remaining P100,000.
d. It shall pay a tax of one-half of 1% of the P2 million gross
sales.
55. The payor of passive income subject to final tax is required to
withhold the tax from the payment due the recipient. The
withholding of the tax has the effect of
a. A final settlement of the tax liability on the income.
b. A credit from the recipient's income tax liability.
c. Consummating the transaction resulting in an income.
d. A deduction in the recipient's income tax return.
56. Winterfell, Inc., bought a parcel of land in 2015 for P7 million
as part of its inventory of real properties. In 2017, it sold the
land for P12 million which was its zonal valuation. In the same
year, it incurred a loss of P6 million for selling another parcel
of land in its inventory. These were the only transactions it had
in its real estate business. Which of the following is the
applicable tax treatment?
a. Winterfell shall be subject to a tax of 6% of P12 million.
b. Winterfell could deduct its P6 million loss from its P5
million gain.
c. Winterfell’s gain of P5 million shall be subject to the
holding period.
d. Winterfell's P6 million loss could not be deducted from its
P5 million gain.
Downloaded by John Paul Magbitang (johnpaulmagbitang04@gmail.com)
lOMoARcPSD|15752181
57. Passive income includes income derived from an activity in which
the earner does not have any substantial participation. This type
of income is
a. Usually subject to a final tax.
b. Exempt from income taxation.
c. Taxable only if earned by a citizen.
d. Included in the income tax return.
58. In 2017, Alice earned P500,000 as income from her beauty parlor
and received P250,000 as Christmas gift from her aunt. She had no
other receipts for the year. She spent P150,000 for the operation
of her beauty parlor. For tax purposes, her gross income for 2017
is
a. P750,000 c. P350,000
b. P500,000 d. P600,000
59. Which of the following items is not part of gross income to be
reported in the income tax return?
a. Increase in value of land
b. Gambling winnings
c. Prize of P10,000
d. Gain from sale of store’s air conditioner
60. Mr. Yu leased his lot to Mr. Uy. The contract calls for Mr. Uy to
construct a house which would serve as the residence of the
latter, the ownership thereof to be vested in Mr. Yu after the
expiration of the lease. When the house was completely
constructed, the remaining term of the lease was 10 years. The
residential house had an estimated useful life of 15 years.
What is the tax implication of the leasehold improvement?
a. Mr. Yu derives taxable income on the improvement; Mr. Uy can
claim depreciation expense as a deduction from gross income.
b. Mr. Yu derives taxable income on the improvement; Mr. Uy
cannot claim depreciation expense as a deduction from gross
income.
c. Mr. Yu does not derive taxable income on the improvement;
Mr. Uy cannot claim depreciation expense as a deduction from
gross income.
d. Mr. Yu does not derive taxable income on the improvement;
Mr. Uy can claim depreciation expense as a deduction from
gross income.
61. Assume the same facts in the immediately preceding number, except
that at the time of the completion of the residential house, the
remaining term of the lease was 15 years while the useful life of
the house was 10 years.
What is the tax implication of the leasehold improvement?
a. Mr. Yu derives taxable income on the improvement; Mr. Uy can
claim depreciation expense as a deduction from gross income.
b. Mr. Yu derives taxable income on the improvement; Mr. Uy
cannot claim depreciation expense as a deduction from gross
income.
Downloaded by John Paul Magbitang (johnpaulmagbitang04@gmail.com)
lOMoARcPSD|15752181
c. Mr. Yu does not derive taxable income on the improvement;
Mr. Uy cannot claim depreciation expense as a deduction from
gross income.
d. Mr. Yu does not derive taxable income on the improvement;
Mr. Uy can claim depreciation expense as a deduction from
gross income.
62. Which of the following expenses may be deducted from gross
compensation income?
a. Depreciation of permanent assets
b. Premium payments on health and/or hospitalization insurance
c. Bad debts written off
d. Optional standard deduction
63. Which of the following items of interest expense may be deducted
from gross income?
a. Interest on corporation’s preferred stock
b. Interest on loan for construction of a rest house
c. Interest for delinquency in the payment of percentage tax
d. Interest on bank loan to finance petroleum exploration
64. Which of the following taxes may be deducted from gross income?
a. Percentage tax on sale of listed stock
b. Business permit fee paid to the city government
c. Income tax
d. Tax on interest on bank deposit
65. Who among the following taxpayers may not claim a tax credit or
deduction on income tax paid to foreign countries?
a. Resident citizens
b. Resident aliens
c. Domestic corporations
d. General Co-Partnerships
66. The loss from sale or exchange of property is deductible from
gross income where the sale or exchange is:
a. Between fiduciary of a trust and the fiduciary of another
trust if they have the same grantor
b. Between fiduciary of a trust and the beneficiary of such
trust
c. Between an individual and his first cousin
d. Between an individual and a corporation if the former owns
more than 50% in value of the outstanding capital stock of
the latter
67. Anne, claimed a bad debt of P50,000 as a deductible expense in the
taxable year 2016. In 2017, Anne was able to recover the P50,000
debt already written off in the preceding year.
What is the treatment for tax purposes of the recovery of the bad
debt?
a. Report the recovery of the bad debt as gross income in 2016.
b. Report the recovery of the bad debt as gross income in 2017.
Downloaded by John Paul Magbitang (johnpaulmagbitang04@gmail.com)
lOMoARcPSD|15752181
c. Disregard the recovery of the bad debt.
d. Amend the 2016 income tax return to rectify the deduction
for bad debt claimed.
68. Which of the following assets shall be subject to depletion?
a. Machinery
b. Land containing ore deposit
c. Commercial
d. Goodwill
69. Which of the following individual taxpayers may claim basic and
additional personal exemptions for income tax purposes?
a. Non-resident aliens engaged in trade or business in the
Philippines, in the absence of reciprocity
b. Non-resident aliens not engaged in trade or business in the
Philippines
c. Both (a) and (b)
d. Neither (a) nor (b)
70. Harry works as financial consultant in an oil firm in Dubai. Aside
from his salary thereat, he also maintains a 10-door apartment in
Manila which he inherited from his parents when he was already
married. On the other hand, Wilma, his wife, is employed as a loan
officer at a local bank. Data pertaining to their dependents
appear below for the taxable year 2017:
 Anton - Son who turned 23 on April 1, 2017; incapable of
self-support due to loss of both legs in an accident;
 Bunny - 21 year old daughter who is taking up culinary arts
in Paris, France;
 Charlie - 15-year old adulterous son of Harry living with the
couple;
 Dina - 12-year old child who died from a vehicular accident
on January 1, 2017; and
 Evan - 80-year old father of Wilma, supported by her and
living with the couple.
The basic and additional personal exemptions of Harry for the
taxable year 2017 amounts to:
a. P50,000 and P100,000, respectively
b. P50,000 and P75,000, respectively
c. P50,000 and P0, respectively
d. P0 and P0, respectively
71. Assume the same facts above, the basic and additional personal
exemptions of Wilma for the taxable year 2016 amount to:
a. P50,000 and P100,000, respectively
b. P50,000 and P75,000, respectively
c. P50,000 and P0, respectively
d. P0 and P0, respectively
Downloaded by John Paul Magbitang (johnpaulmagbitang04@gmail.com)
lOMoARcPSD|15752181
72. Which of the following statements does not characterize a capital
asset?
a. It may be real or personal property.
b. It is not always subject to a holding period.
c. It is normally subject to value-added tax when it is sold.
d. It is not always subject to a final tax.
73. Which of the following transactions is exempt from capital gains
tax?
a. The sale of the principal residence of the taxpayer where
the entire proceeds is used to purchase a vacation lot at
Tagaytay
b. The sale of a beach lot of the taxpayer where the entire
proceeds is used to construct his principal residence
c. The sale of the principal residence of the taxpayer for the
second time in ten (10) years to purchase another principal
residence
d. None of the choices
74. Which of the following transactions is treated as a capital asset
transaction for income tax purposes?
a. Sale of a residential lot by a subdivision developer
b. Sale of a used delivery truck by a retailing company
c. Liquidation of partnership business
d. Sale of shares of stock by a dealer in securities
75. Which of the following is not an attribute of a deferred-payment
sale?
a. The initial payments exceed 25% of the selling price in the
year of sale.
b. The obligations or promissory notes received by the vendor
from the vendee are considered as equivalent to cash.
c. The tax may be paid in installments.
d. The sale involves both real and personal property.
76. The deductible expenses of an estate may consist of:
a. Deductible expenses allowed to an individual taxpayer
b. Income distributed to beneficiaries
c. Both (a) and (b)
d. Neither (a) nor (b)
77. Determine which of the following trusts shall the taxable income
be consolidated and the income tax thereon computed on the basis
of such consolidated income?
a. Trust No. 1 and Trust No. 2 have the same grantor and with
different beneficiaries.
b. Trust No. 1 and Trust No. 2 have the same grantor and the
same beneficiary.
c. Trust No. 1 and Trust No. 2 have different grantors and the
same beneficiary.
d. Trust No. 1 and Trust No. 2 have different grantors and the
same fiduciary and beneficiary.
Downloaded by John Paul Magbitang (johnpaulmagbitang04@gmail.com)
lOMoARcPSD|15752181
78. Inday is a resident citizen of the Philippines. Data for a year:
Gross income from business P 700,000
Royalty from books 40,000
Gain on direct sale to buyer of
shares of stock of a domestic corporation
held as capital asset 70,000
Loss on sale of land in the Philippines held
as capital asset with cost of P1,500,000 when the
zonal value is P1,200,000 500,000
Business Expenses 300,000
How much is the total income tax expense for the year?
a. P177,500 c. P159,500
b. P80,000 d. P156,000
Gross Income from Business 700,000
Less: Business Expense 300,000
Personal Exemptions 50,000 350,000
Net Taxable income 350,000
Tabular Schedule:
1st 250,000 Tax is 50,000
Excess of 250,000 - 350,000 = 100,000 x 30% 30,000
Capital Gains Tax on Shares (70,000 x 5%) 3,500
Royalty Income (40,000 x 10%) 4,000
Loss on Sale (Zonal Value 1.2 x 6%) 72,000
Total Income Tax Expense 159,500
79. Mercy is a citizen and resident of the Philippines. She had a
compensation income (net of exclusions) of P200,000 and a net
income from business of P700,000 for a year. She made quarterly
income tax payments amounting to P237,000 and her employer
withheld P25,000 on her compensation income.
The income tax payable (refundable) for the year is:
a. (P25,000) c. (P42,000)
b. P237,000 d. (P37,500)
Compensation Income 200,000
Net Income from Business 700,000
Total Income 900,000
Basic Exemptions (50,000)
Taxable Income 850,000
1st 500,000 Tax 125,000
Excess of 500,000- 850,000 x 32% 112,000
Total Tax Due 237,000
Less: Taxes Payments
Quarterly Payments 237,000
Compensation CWT 25,000 262,000
Refundable /Creditable Tax (25,000)
Downloaded by John Paul Magbitang (johnpaulmagbitang04@gmail.com)
lOMoARcPSD|15752181
80. EMT has the following data on his passive income earned during the
year 2016:
Philippines Abroad
Interest income from bank deposits 45,000 25,000
Interest income from FCDU 50,000 -0-
Royalties from books 20,000 30,000
Royalties from computer programs 20,000 40,000
Dividend income from a domestic corporation 27,000 13,000
Dividend income from a foreign corporation 33,000 22,000
How much is the final withholding tax if the taxpayer is a
resident citizen?
a. P21,450 c. P17,700
b. P20,400 d. P36,250
Philippines
Interest income from bank deposits 45,000 x 20% = 9,000
Interest income from FCDU 50,000 x 7.5% = 3,750
Royalties from books 20,000 x 10% = 2,000
Royalties from computer programs 20,000 x 20% = 4,000
Dividend income from a
domestic corporation 27,000 x 10% = 2,700
Total Withholding Taxes 21,450
81. How much is the final withholding tax if the taxpayer is a
resident alien?
a. P21,450 c. P17,700
b. P20,400 d. P36,250
SAME COMPUTATION AS RESIDENT CITIZEN
82. How much is the final withholding tax if the taxpayer is a non-
resident citizen?
a. P21,450 c. P17,700
b. P20,400 d. P36,250
Philippines
Interest income from bank deposits 45,000 x 20% = 9,000
Royalties from books 20,000 x 10% = 2,000
Royalties from computer programs 20,000 x 20% = 4,000
Dividend income from a domestic
corporation 27,000 x 10% = 2,700
Total Withholding Taxes 17,700
83. How much is the final withholding tax if the taxpayer is a non-
resident alien engaged in trade or business?
a. P21,450 c. P17,700
b. P20,400 d. P36,250
Downloaded by John Paul Magbitang (johnpaulmagbitang04@gmail.com)
lOMoARcPSD|15752181
Philippines
Interest income from bank deposits 45,000 x 20% = 9,000
Royalties from books 20,000 x 10% = 2,000
Royalties from computer programs 20,000 x 20% = 4,000
Dividend income from a domestic
corporation 27,000 x 20% = 5,400
Total Withholding Taxes 20,400
84. How much is the final withholding tax if the taxpayer is a non-
resident alien not engaged in trade or business?
a. P21,450 c. P17,700
b. P20,400 d. P36,250
Interest income from bank deposits 45,000
Royalties from books 20,000
Royalties from computer programs 20,000
Dividend income from a domestic corporation 27,000
Dividend income from a foreign corporation 33,000
TOTAL PASSIVE INCOME 145,000
Final Withholding Tax Rate 25%
Total Withholding Taxes 36,250
85. Nonoy is an employee of a firm in Quezon City. He is supporting
his 4 year old brother who is living with him. Data on his
compensation income for the year shows:
Regular Salary P 240,000
Thirteenth month pay 20,000
Quarterly bonus 40,000
Payroll Deductions:
SSS Premiums 3,000
Philhealth contributions 1,200
Pagibig contributions 4,000
Labor union dues 1,000
Premium payments on hospitalization insurance 3,000
Payment of loan 5,000
86. How much is the taxable income?
a. P210,800 c. P208,400
b. P207,800 d. P260,800
Regular Salary P 240,000
Total Income 240,000
Less: Basic Exemption (50,000)
SSS Premiums (3,000)
Philhealth contributions (1,200)
Pagibig contributions (4,000)
Labor union dues (1,000)
Net Taxable Income 180,800 +30,000 if old laws = 218,000
Downloaded by John Paul Magbitang (johnpaulmagbitang04@gmail.com)
lOMoARcPSD|15752181
87. A domestic corporation, in its fifth (5th
) year of operations, had
the following data for the year:
Net sales P 2,000,000
Capital gain on direct sale to a buyer
of shares of a domestic corporation for P500,000 200,000
Capital gain on sale thru a real
estate broker of land and building outside
the Philippines for P5,000,000 1,000,000
Dividend from a domestic corporation 50,000
Interest on bank deposit 40,000
Cost of sales 600,000
Quarterly corporate income tax paid 190,000
Operating expenses 500,000
The income tax payable upon filing of the annual income tax return
is:
a. P425,000 c. P570,800
b. P380,000 d. P520,800
Net Sales 2,000,000
Cost of Sales (600,000)
Operating Expenses (500,000)
Net Income from Ordinary Business 900,000
Add Other Income Broker Outside Phils 1,000,000
Total Taxable Income 1,900,000
Corporate Tax Rate 30%
Income Tax Due 570,000
Less Quarterly Income Tax Paid (190,000)
Income Tax Due and Payable 380,000
88. The Kultura Foundation of the Philippines, a non-stock, non-profit
corporation, organized and operated exclusively to preserve and
show-case Philippine cultural practices, music, dances, and folk
arts, deriving funding from mostly donations, had the following
data for the year:
Donations received P 20,000,000
Interest income from bank deposit 100,000
Rent income from properties received as donation
(net of 5% withholding tax) 475,000
Expenses related to its rent income 30,000
89. How much is the income tax expense of the Corporation for the
year?
a. P161,000 c. P172,000
b. P136,000 d. P125,600
Rent Income (Gross Up) 500,000
Less: Expenses (30,000)
Net Rent Income 470,000
Downloaded by John Paul Magbitang (johnpaulmagbitang04@gmail.com)
lOMoARcPSD|15752181
Tax Rate 30%
Tax Expense 141,000
Add Interest Income Tax 20,000
Total Income Tax Expense 161,000
90. A tax imposed in the nature of a penalty to the corporation to
deter tax avoidance of shareholders who avoid paying the dividends
tax on the earnings distributed to them by the corporation.
a. Minimum corporate income tax
b. Optional corporate income tax
c. Improperly accumulated earnings tax
d. Capital gains tax
91. First statement: An accumulation of earnings or profits (including
undistributed earnings or profits of prior years) is unreasonable
if it is not necessary for the purpose of the business,
considering all the circumstances of the case.
Second statement: The term "reasonable needs of the business" are
hereby construed to mean the immediate needs of the business,
including reasonably anticipated needs.
a. Only the first statement is correct
b. Only the second statement is correct
c. Both statements are correct
d. Both statements are incorrect
92. The Improperly Accumulated Earnings Tax (IAET) is imposed on
improperly accumulated taxable income earned starting January 1,
1998 by domestic corporations as defined under the Tax Code and
which are classified as closely-held corporations at the rate of:
a. twenty percent (20%). c. ten percent (10%).
b. fifteen percent (15%). d. five percent (5%).
93. The Improperly Accumulated Earnings Tax shall not apply to which
of the following corporations?
a. Banks and other non-bank financial intermediaries
b. Insurance companies
c. Publicly-held corporation
d. All of the choices
94. First statement: Once the profits have been subjected to
improperly accumulated earnings tax, the same shall no longer be
subject to the same tax in later years even if not declared as
dividends.
Second statement: Profits which have been subjected to improperly
accumulated earnings tax when finally declared as dividends shall
be subject to tax on dividends.
a. Both statements are true
b. Both statements are false
c. Only the first statement is true
d. Only the second statement is true
Downloaded by John Paul Magbitang (johnpaulmagbitang04@gmail.com)
lOMoARcPSD|15752181
95. To avoid payment of IAET, when must the dividends be declared and
paid or issued?
a. Not later than one year following the close of the taxable
year
b. Not later than the 15th day following the close of the
taxable year
c. Not later than 60th day following the close of the taxable
quarter
d. None of the choices
TAX REMEDIES
96. Rosalie, a compensation income earner, filed her income tax return
for the taxable year 2013 on March 30, 2014. On May 20, 2017,
Rosalie received an assessment notice and letter of demand
covering the taxable year 2013 but the postmark on the envelope
shows April 10, 2017. Her return is not a false and fraudulent
return. Can Rosalie raise the defense of prescription?
a. No. The 3 year prescriptive period started to run on April
15, 2014, hence, it has not yet expired on April 10, 2017.
b. Yes. The 3 year prescriptive period started to run on April
15, 2014, hence, it had already expired by May 20, 2017.
c. No. The prescriptive period started to run on March 30,
2014, hence, the 3 year period expired on April 10, 2017.
d. Yes. Since the 3-year prescriptive period started to run on
March 30, 2014, it already expired by May 20, 2017.
97. On March 30, 2012 Emmett Foods, Inc. received a notice of
assessment and a letter of demand on its April 15, 2009 final
adjustment return from the BIR. Emmett Foods then filed a request
for reinvestigation together with the requisite supporting
documents on April 25, 2012. On June 2, 2012, the BIR issued a
final assessment reducing the amount of the tax demanded. Emmett
Foods was satisfied with the reduction, it did not do anything
anymore. On April 15, 2017 the BIR garnished the corporation's
bank deposits to answer for the tax liability. Was the BIR action
proper?
a. Yes. The BIR has 5 years from the filing of the protest
within which to collect.
b. Yes. The BIR has 5 years from the issuance of the final
assessment within which to collect.
c. No. The taxpayer did not apply for a compromise.
d. No. Without the taxpayer’s prior authority, the BIR action
violated the Bank Deposit Secrecy Law.
98. Renesmee, Inc. received a notice of assessment and a letter from
the BIR demanding the payment of P3 million pesos in deficiency
income taxes for the taxable year 2015. The financial statements
of the company show that it has been suffering financial reverses
from the year 2016 up to the present. Its asset position shows
that it could pay only P500,000.00 which it offered as a
compromise to the BIR. Which among the following may the BIR
Downloaded by John Paul Magbitang (johnpaulmagbitang04@gmail.com)
lOMoARcPSD|15752181
require to enable it to enter into a compromise with Renesmee,
Inc.?
a. Renesmee must show it has faithfully paid taxes before 2016.
b. Renesmee must promise to pay its deficiency when financially
able.
c. Renesmee must waive its right to the secrecy of its bank
deposits.
d. Renesmee must immediately deposit the P500,000 with the BIR.
99. As a rule, within what period must a taxpayer elevate to the Court
of Tax Appeals a denial of his application for refund of income
tax overpayment?
a. Within 30 days from receipt of the Commissioner’s denial of
his application for refund.
b. Within 30 days from receipt of the denial which must not
exceed 2 years from payment of income tax.
c. Within 2 years from payment of the income taxes sought to be
refunded.
d. Within 30 days from receipt of the denial or within two
years from payment.
100. What is the effect on the tax liability of a taxpayer who does not
protest an assessment for deficiency taxes?
a. The taxpayer may appeal his liability to the CTA since the
assessment is a final decision of the Commissioner on the
matter.
b. The BIR could already enforce the collection of the
taxpayer's liability if it could secure authority from the
CTA.
c. The taxpayer's liability becomes fixed and subject to
collection as the assessment becomes final and collectible.
d. The taxpayer's liability remains suspended for 180 days from
the expiration of the period to protest.
101. The taxpayer seasonably filed his protest together with all the
supporting documents. It is already July 31, 2017, or 180 days
from submission of the protest but the BIR Commissioner has not
yet decided his protest. Desirous of an early resolution of his
protested assessment, the taxpayer should file his appeal to the
Court of Tax Appeals not later than
a. August 31, 2017.
b. August 30, 2017.
c. August 15, 2017.
d. August 1, 2017.
Downloaded by John Paul Magbitang (johnpaulmagbitang04@gmail.com)
lOMoARcPSD|15752181
DOCUMENTARY STAMP TAX
102. A newly formed corporation issued shares of stocks to its
incorporators for P150,000. The par value of the shares issued is
P100,000. How much is the documentary stamp tax?
a. P500 c. P750
b. P1,000 d. P1,500
Par Value Divide by 200 x 1 peso = 100,000 / 200 = 500
103. Mr. T invested in shares of stock of Kapisananngmga Sisters Inc.
amounting to P100,000 with par value of P80,000. After 2 years, he
disposed said shares directly to Mr. B for P230,000.
The documentary stamp tax on above transaction is:
a. P400 c. P300
b. P1,150 d. P862.50
**80,000 / 200 = 400 x 75% = 300
104. Based on number 97, but assuming the shares are without par value,
how much is the documentary stamp tax?
a. P500 c. P750
b. P1,000 d. P0.00
**Selling Price - Cost = 230,000 - 80,000 = 150,000 / 200 = 750
105. Continuing number 99, if the shares were subsequently sold for
P200,000, how much is the documentary stamp tax?
a. P750 c. P1,500
b. P187.50 d. P375
**Doc Stamp Upon Original Issuance 750 x 25% = 187.50
106. Mr. Purisima owns a resthouse in Pampanga acquired by him for
P10,750,000. He sold the same to Mr. Apo for P5,000,000. The fair
market value at the time of sale per assessor’s office is
P10,000,000 while zonal value is P15,000,000. The documentary
stamp tax on the transaction is:
a. P75,000 c. P150,000
b. P225,000 d. P161,250
**15,000,000/1000 = 15,000 x 15 pesos = 225,000
107. Who is liable to the payment documentary stamp tax?
a. Mr. Purisima
b. Mr. Apo
c. It depends on the agreement of the parties
d. It depends on who is benefiting on the transaction
Downloaded by John Paul Magbitang (johnpaulmagbitang04@gmail.com)
lOMoARcPSD|15752181
TAXATION QUIZZER PART 2
1. The BIR is assessing deficiency withholding tax on total Repairs and
Maintenance claimed by non-stock non-profit organization. The BIR
alleges that foundation failed to withhold the correct amount of the
2% expanded withholding tax due on the Repairs and
Maintenance expense. As a CPA assisting the foundation in the BIR tax
investigation, you can reason that ___.
a. The BIR should have considered the 1% expanded withholding tax
instead of the 2%in determining the deficiency tax.
b. The foundation is exempt from the tax and hence is not liable to
the deficiency withholding tax.
c. The foundation is exempt from tax and hence is not required to
withhold tax on its expenses.
d. The BIR should have considered both the 1% and 2% withholding tax
in determining the deficiency tax.
2. Which of the following statement is true?
a. Taxes are in the nature if contracts between the taxpayers and
the government.
b. Taxes and debts are similar nature and character.
c. As the general rule, no set-off is allowed against the demands
for taxes levied for general or local government purposes.
d. In taxation, the personal consent of the individual taxpayers is
required.
3. Which of the following is not an attribute of tax?
a. It is an enforced contribution on a person taxed.
b. It is levied by the executive department of the state.
c. It is imposed by the state which has jurisdiction over the
person, property, or excises.
d. It is generally in money
4. Which of the statement below is grammatically correct?
a. The imposition of minimum corporate income tax may be suspended
if substantial losses are sustained due to a prolonged labor
dispute.
b. The imposition of minimum corporate income tax maybe suspend if
substantial losses are sustained due to prolonged labor dispute.
c. The imposition of minimum corporate income tax may be suspended
if substantial losses are sustained due to a prolong disputes.
d. The imposition of minimum corporate income tax may be suspended.
If substantial loss are sustained due to a prolonged labor
dispute.
5. Which of the following statements is correct?
a. If what is delegated is tax administration, the delegation is
invalid.
b. If what is delegated is tax administration, the delegation is
valid.
c. Tariff powers cannot be delegated to the President.
d. As a general rule, taxation can be further delegated.
Downloaded by John Paul Magbitang (johnpaulmagbitang04@gmail.com)
lOMoARcPSD|15752181
6. The BIR issued a tax assessment against the taxpayer who was not given
sufficient time to protest the said assessment. The taxpayer noted
that their competitors were issued tax assessments but were given
enough time to protest. The BIR violated the _____________ of the
constitution.
a. Equal protection clause c. Equitability principle
b. Due process clause d. Uniformity principle
Items 7-10 are based on the following information:
On March 30, 2012, XXX, Inc., received a notice of assessment and a letter
of demand on its April 15, 2009 final adjustment return from the BIR.
XXX, Inc., then filed a request for reinvestigation together with the
requisite supporting documents on April 25, 2012. On June 2, 2012, the
BIR issued a final assessment reducing the amount of the tax demand.
Since XXX, Inc., was satisfied with the reduction, it did not do
anything anymore. On April 15, 2017 the BIR garnished the
corporation’s bank deposits to answer the liability.
7. Was the BIR action proper?
a. No, the taxpayer did not apply for the compromise.
b. Yes, the BIR has 5years from the filing within which to collect.
c. No, without the taxpayer’s prior authority, the BIR action
violated the Bank Deposit Secrecy Law.
d. Yes, the BIR has 5 years from the issuance if the final
assessment within which to collect.
8. What is the effect of the XXX, Inc.’s failure to file a protest on its
assessed deficiency taxes?
a. The taxpayer may file a motion for reconsideration to the CIR on
the matter.
b. The taxpayer may appeal his liability to the CTA since the
assessment is a final decision of the Commissioner on the matter.
c. The taxpayer’s liability becomes fixed and subject to collection
as the assessment becomes final and collectible.
d. The BIR could already enforced the collection of the taxpayer’s
liability if it could secure authority from the CTA.
9. Which statement is correct? The collection of a deficient tax
assessment by distraint and levy:
a. May be done only once during the taxable year.
b. Must be done successively, first by distrait and then by levy;
c. May be repeated, if necessary, until the full amount due,
including all expenses, is collected;
d. Automatically covers the bank deposits of a delinquent taxpayer.
10. Which of the following is grammatically correct?
a. Tax assessment refer to the process of determining the correct
amount of tax due in accordance with the prevailing tax laws.
b. Tax assessment refer to the process of determining the correct
amount of tax due in accordance with the prevailed tax laws.
Downloaded by John Paul Magbitang (johnpaulmagbitang04@gmail.com)
lOMoARcPSD|15752181
c. Tax assessment refers to the process of determining the correct
amount of tax due in accordance with the prevailed tax laws.
d. Tax assessment refers to the process of determining the correct
amount of tax due in accordance with the prevailing tax laws.
11. A joker was commissioned in a kiddy party to perform magic. The comic
was to be paid P100, 000 for his performance and the parties signed
the necessary contract. He then gratuitously assigned his rights under
the contract to his son. The son later on collected the P100,
000 talent fee of his father from the contractee. The national
internal revenue tax/es payable is/are:
a. Income tax only.
b. Donor’s tax only.
c. Both income and donor’s taxes.
d. Neither income tax nor donor’s tax.
12. Which one of the following statements is wrong?
a. Income out of the labor of the wife is conjugal property.
b. Income out of the separate property of the husband is conjugal
property.
c. Amount receivable as retirement benefit under R.A. No. 4917
during the marriage is conjugal property.
d. Property received is donation when the fair value was P2,
000,000, resulting in a gain of P500, 000. The gain is conjugal
property.
13. Amount receivable by the estate of the deceased, his executor or
administration as a beneficiary under life insurance policy taken by
the decedent upon his own life is:
a. Excluded from gross state.
b. Part of gross state if the beneficiary is revocable.
c. Excluded from gross state if the beneficiary is irrevocable.
d. Part of gross state whether the beneficiary is revocable or
irrevocable.
14. The following journal entry was made in the purchases journal of a
VAT-registered taxpayer:
Purchases xxx
Cash or Accounts Payable xxx
The journal entry signifies that:
a. Purchases were from a non-VAT supplier.
b. Purchases were from a VAT-registered supplier.
c. Purchases were either from non-VAT or VAT-registered supplier.
d. Input tax has been taken as part of the cost of purchase.
15. A common carrier by land is engaged in the transport of passengers,
goods and cargoes. He is not VAT-registered. What business tax or
taxes is he liable to pay?
a. 12% value-added tax
b. 3% common carrier’s tax
Downloaded by John Paul Magbitang (johnpaulmagbitang04@gmail.com)
lOMoARcPSD|15752181
c. 12% VAT on gross receipts from transport of goods and cargoes and
3% common carrier’s tax on gross receipts from transport of
passengers.
d. 3% tax on VAT-exempt persons on gross receipts from transport of
passengers.
16. Statement 1. As to the property of the state, exemption is the rule
and taxation the exception.
Statement 2. As of the property of the taxpayer, taxation is the rule and
exemption the exception.
a. Both statements are correct
b. Both statements are incorrect.
c. Statement 1 is correct while statement 2 is incorrect.
d. Statement 1 is incorrect while statement 2 is correct.
17. Foreign income taxes paid by the resident citizen or domestic
corporation.
a. May be claimed only as tax credit.
b. May be claimed only as tax deduction.
c. Do not qualify either as a tax credit or as a tax deduction.
d. May be claimed either as a tax credit or as a tax deduction at
the option of the income taxpayer.
18. Statement 1. Government agencies performing essential governmental
functions are subject to tax unless expressly exempted.
Statement 2. Government agencies performing propriety function are exempt
from tax unless expressly taxed.
a. Both statements are correct.
b. Both statements are incorrect.
c. Statement 1 is true while statement 2 is false.
d. Statement 2 is true while statement 1 is false.
19. Statement 1. Tax avoidance or tax minimization is the use by the
taxpayer of legally permissible methods in order to reduce tax
liability.
Statement 2. Tax evasion or tax dodging is the use by the taxpayer of
illegal means to defeat or lessen the payment of tax.
a. Both statements are correct.
b. Both statements are incorrect.
c. Statement 1 is correct while statement 2 is incorrect.
d. Statement 2 is correct while statement 1 is incorrect.
20. Which one among the following items below is included in the gross
state?
a. Revocable transfer.
b. Transfer with reservation of certain rights.
c. Transfer under general power of appointment.
d. Transfer in contemplation of death which is onerous.
Downloaded by John Paul Magbitang (johnpaulmagbitang04@gmail.com)
lOMoARcPSD|15752181
21. Which of the following statement is correct?
a. The final tax on compensation of special kind of non-resident
aliens is 25% of the gross income.
b. Interest income from a foreign currency deposit unit in the
Philippines of a non resident alien is not subject to final tax.
c. Informer’s reward is subject to final tax of 10% based on the 10%
of the value of tax assessed or P1, 000,000 whichever is higher.
d. Prizes exceeding P10,000 derived by non-resident alien not engage
in trade or business here in the Philippines is subject to a
final tax of 20%
22. The power to decide disputed assignment, refunds of internal revenue
taxes, fees or other charges, penalties imposed in relation thereof,
or other matters arising other the tax code or other laws or portions
thereof administered by the BIR is vested in the:
a. Secretary of Finance
b. Commissioner of Internal Revenue
c. Court of Tax Appeals
d. Regional Trial Court
23. The commissioner of the Internal Revenue (CIR) is prohibited by law to
look into the bank accounts of taxpayer, except when:
a. Taxpayer is accused of heinous crime
b. Taxpayer did not invoke his right to privacy during the tax
audit.
c. Taxpayer applies for compromise on tax obligation on account of
financial incapacity.
d. The CIR has reason to believe that taxpayer has filed a false or
fraudulent return.
24. A VAT-registered realty company sells real property in the course of
its business. On April 30, 2016, it has sold a lot under the following
items (VAT excluded).
Selling Price P4,000,000
Down payment, 4/30/16 400,000
1st Installment, 4/30/17 600,000
2nd Installment, 4/30/18 1,000,000
3rd Installment, 4/30/19 1,000,000
4th Installment, 4/30/20 1,000,000
Interest and other charges of 10%
On unpaid balance per installment
The zonal value of the lot at the time of sale is P4, 800,000.
The output VAT for the installment received on April 30, 2017, is:
a. P43,200
b. P86,400
c. P115,200
d. P129,600
Downloaded by John Paul Magbitang (johnpaulmagbitang04@gmail.com)
lOMoARcPSD|15752181
Output Tax (4,800,000 x 12%) = 576,000
Principal 600,000 / 4,000,000 x 576,000 = 86,400
Interest 3,600, 000 x 10% x 12% = 43,200
Total 129,600
25. Counting No. 24, the output VAT on April 30, 2020, is:
a. P132, 000
b. P144,000
c. P156,000
d. P228,000
4,800,000 x 12% = 576,000 out put tax
Principal 1,000,000 / 4,000,000 x 576,000 = 144,000
Interest 1,000,000 x 10% x 12% = 12,000
Total 156,000
26. Assuming that the real property in No. 24 is the residential lot sold
for cash of P1, 750,000 (VAT not separately blend in the sales
document) on April 30, 2016. Zonal value of the lot at the time of
sale is P1, 900,000. The output VAT on the sale is:
a. P187,500
b. P210, 000
c. P228,000
d. None, as it is exempt the VAT.
27. The lease of real or personal property is subject to:
i. 7% gross receipt tax (GRT) if the lessor is the bank.
ii. 3% percentage tax or 12% value-added tax (VAT) if the
lessor is not a bank.
a. No to I and II
b. Yes to I and II
c. Yes to I only
d. Yes to II only
28. Ayala Land, Inc. (ALI) bought a parcel of land in 2014 for P7 million
as part of its inventory of real properties. In 2016, it sold the land
for P12 million which was its zonal valuation. In the same year, it
suffered a loss of P6 million for selling another parcel of land from
its inventory. These were the only transactions ALI had in its real
estate business. Which of the following is the applicable tax
treatment?
a. ALI shall be subject to a tax of 6% of P12 million.
b. ALI’s gain of P5 million shall be subject to holding period.
c. ALI could deduct its P6 million loss from its P5 million gain.
d. ALI’s P6 million loss could not be deducted from its P5 million
gain.
29. This is an inherent limitation on the power of taxation:
a. The rule of taxation shall be uniform and equitable.
b. No law impairing the obligations of contacts shall be enacted.
Downloaded by John Paul Magbitang (johnpaulmagbitang04@gmail.com)
lOMoARcPSD|15752181
c. Tax laws can not apply to the property of foreign governments.
d. Charitable institutions, churches, parsonages, convents and all
lands, buildings and improvements, actually, directly, and
exclusively used for religious, charitable or educational purposes
shall be exempt for taxation.
30. X took a life insurance policy of P5 million where the monthly
premium is P10, 000. The proceeds will be paid to X after 25 years
to the X’s estate should X die before completing the equivalent of
25 years payment. If the X outlived the policy, which of the
following is correct?
a. The proceeds will be part of X’s gross estate.
b. The proceeds will be part of X’s taxable income.
c. The proceeds will be party taxable estate and partly exempt.
d. The proceeds will be party taxable income and partly exempt.
31. Containing the preceding number, except that after paying the
equivalent of ten years premium, X transferred the policy to Y for
P1.5 million and Y continued paying the monthly premium as they
mature. After 10 years, X died. Which of the following is correct?
a. The proceeds will be part of X’s gross estate.
b. The proceeds received by Y is part of his taxable income.
c. The amount received by X from Y is part of X’s taxable income.
d. The amount received by the X from Y and the proceeds received by Y
are partly taxable income and partly exempt.
32. An owner of several warehouses for rent, which used to be VAT-
exempt because its annual gross receipts never exceeded P1,919,500
decided to register under the VAT system on January 2, 2016. The
following data were from the first quarter ending March 31, 2016:
Rental from warehousing services, net of VAT P 672,000
Purchases of supplies, gross of VAT 224,000
Inventory of supplies, January 1, 2016 201,600
Actual VAT paid on the inventory of
supplies, January 1, 2016 21,600
The value added tax payable for the quarter is:
a. P23,520
b. P35,040
c. P41,088
d. P52,608
VAT on rental (672,000 x 12 %) 80,640
VAT on purchase of supplies (224,000 x 12/112) (24,000)
Transitional Input Tax(higher) (21,600)
VAT Payable 35,040
Downloaded by John Paul Magbitang (johnpaulmagbitang04@gmail.com)
lOMoARcPSD|15752181
33. Assuming that one vacant warehouse in the preceding number was
rented out for the whole month of April 2016 and received P107, 000
as rent, gross of VAT but net of the applicable creditable
withholding income tax (expanded). The output VAT on the rental
receipt is:
a. P12,000
b. P12,240
c. P12,840
d. P13,440
107,000/107% = 100,000 x 12% = 12,000
Numbers 34 to 42 are based on the following information:
Dina Cabangon, a citizen and resident of the Philippines, died on November
1, 2016. Her marriage was under the system of absolute community of
property. The following properties and obligations were left:
Property received by Dina as inheritance on February 1, 2015
(during the marriage) P2,000,000
Real property acquired through the labor of both Dina and
her Husband during the marriage (family home) 4,000,000
Property owned by Dina before marriage 300,000
Property owned by Dina’s Husband before marriage 200,000
Funeral expenses 300,000
Unpaid mortgage on property inherited 200,000
Judicial expenses for the settlement of the estate 80,000
Unpaid obligations (excluding the unpaid mortgage) 40,000
The property received as inheritance was part of the gross estate of the
prior decedent at a fair market value of P1,100,000. At the time of
inheritance, it was mortgage for P300,000. Dina was able to pay P100,000
before she passed away.
34. The total community property is:
a. P6,500,000 c. P4,000,000
b. P4,500,000 d. P2,300,000
35. The total exclusive property is:
a. P4,200,000 c. P2,300,000
b. P2,500,000 d. P2,000,000
36. The total ordinary community deduction is:
a. P200,000 c. P420,000
b. P320,000 d. P445,000
37. The total ordinary exclusive deduction (excluding vanishing
deduction) is:
a. P400,000 c. P200,000
b. P300,000 d. P100,000
Downloaded by John Paul Magbitang (johnpaulmagbitang04@gmail.com)
lOMoARcPSD|15752181
38. The deduction for family home is:
a. P0 c. P2,000,000
b. P1,000,000 d. P4,000,000
39. The amount of vanishing deduction is:
a. P0 c. P736,000
b. P816,000 d. P656,000
40. The total special deduction is:
a. P0 c.P2,000,000
b. P1,000,000 d.P4,000,000
41. The taxable net estate is:
a. P1,154,000 c. P3,154,000
b. P2,154,000 d. P4,244,000
SOLUTION:
Separate Common Total
2,000,000 4,000,000
300,000
200,000
Gross Estate 2,000,000 4,500,000 6,500,000
Funeral Expenses:
Actual 300,000
Limit (6.5 x 5%) 325,000
Threshold 200,000
Whichever is lower (200,000)
Judicial Expenses (80,000)
Indebtedness (200,000) (40,000)
Vanishing Deductions* (736,000)
Net Estate After
Ordinary Deductions 1,064,000 4,180,000 5,244,000
Family Home (1,000,000)
Standard Deductions (1,000,000)
Share of Surviving Spouse 1/2 x 4,180,000 (2,090,000)
Taxable NE 1,154,000
*Vanishing Deduction:
Lower FMV 1,100,000
Mortgage Assumed and Paid (100,000)
Initial Basis 1,000,000
Pro Rated Deductions:
1,000,000/6,500,000 x 520,000 (80,000)
Final Basis 920,000
Vanishing Rate _ 80%__
Vanishing Deductions 736,000
42. Going back to the original problem, except that the marriage of
Dina Cabangon to her spouse was under the system of conjugal
partnership of gains. The taxable net estate would be:
a. P3,206,032 c. P1,206,032
b. P2,206,033 d. P1,154,000
Downloaded by John Paul Magbitang (johnpaulmagbitang04@gmail.com)
lOMoARcPSD|15752181
Separate Common Total
2,000,000 4,000,000
300,000
Gross Estate 2,300,000 4,000,000 6,300,000
Funeral Expenses:
Actual 300,000
Limit (6.5M x 5%) 325,000
Threshold 200,000
Whichever is lower (200,000)
Judicial Expenses (80,000)
Indebtedness (200,000) (40,000)
Vanishing Deductions** (733,968)
Net Estate After
Ordinary Deductions 1,366,032 3,680,000 5,046,032
Family Home (1,000,000)
Standard Deductions (1,000,000)
Share of Surviving Spouse 1/2 x 4,180,000 (1,840,000)
Taxable NE 1,206,302
**Vanishing Deduction:
Lower FMV 1,100,000
Mortgage Assumed and Paid (100,000)
Initial Basis 1,000,000
Pro Rated Deductions:
1,000,000/6,300,000 x 520,000 (82,540)
Final Basis 917,460
Vanishing Rate 80%
Vanishing Deductions 733,968
43. In March 2016, Imelda, who is fond of jewelries, bough the
following: diamond ring for P750, 000; bracelet for P250, 000;
necklace for P500, 000; and a brooch for P500, 000. Imelda drives
income from the exercise of her profession as a topnotch Interior
Designer. In October 2016. Imelda sold her diamond ring, bracelet
and necklace for only P1.25 million, incurring a loss of P250, 000.
She used the P1.25 million to buy a solo diamond ring in November
2016 which she sold for P1.5 million in September 2017. Imelda had
no other transaction on jewelry in 2017. Which among the following
best describes the tax implications arising from the aforesaid
transactions?
a. Imelda may carry over and deduct her 2016 loss only from her 2017
gain.
b. Imelda may deduct her 2016 loss from both her 2017 professional
income.
c. Imelda may not deduct her 2016 loss from both her 2017 professional
income and her gain.
d. Imelda may carry over and deduct her 2016 loss from her 2017
professional income as well as from her gain.
Downloaded by John Paul Magbitang (johnpaulmagbitang04@gmail.com)
lOMoARcPSD|15752181
44. Taxation could be exercise by the following except one. Which one?
a. Judiciary
b. Legislative
c. Local government unit
d. President of the Philippines, in certain cases.
45. Stages, aspects or processes in taxation.
a. Levy of the tax
b. Collection of the tax.
c. Payment of the tax by the taxpayer.
d. All of the above.
46. Statement 1. Onerous donations are subject to donor’s tax.
Statement 2. Gratuitous donations are not subject to donor’s tax.
a. Both statement s are true
b. Both statements are false
c. Only statement 1 is true but not statement 2.
d. Only statement 2 is true but not statement 1.
47. Which of the following statements are correct?
a. Gift-splitting is a form of tax dodging.
b. The uncle who is the brother of the donor’s mother-in-law is a
non-stranger to the donor for purposes of the donor’s tax.
c. A gift made to a relative in January 2016 is to be added to the
gift made to the same relative in December 2015 in determining
the gift tax.
d. Renunciation by an heir including the surviving spouse of his/her
share in the hereditary estate left by the decedent is subject to
donor’s tax if done in favor of identified heirs to the exclusion
or disadvantage of the other co-heir/s in the hereditary estate.
48. Mistah, single and sales executive of a leading pharmaceutical firm
(RiteMed), received in 2016 the following from his employer:
Salary, net of P267,000 withholding tax P 683,000
Allowances and benefits received:
 Rent paid by RiteMed on the house which Mistah occupies for
residential purposes, net of 5% withholding 129,200
 Entertainment allowance subject to liquidation (P75,000 was duly
receipted in the name of RiteMed and used to entertain RiteMed’s
customers and the balance of P25,000 was used to purchase a late
model mobile phone for the personal use of Mistah) 100,000
 Reimbursement of entertainment expenses paid by Mistah (P17,500
was used to entertain Mistah’s boyhood pals and the balance of
P22,500 was used to promote RiteMed’s businesses.)
40,000
Downloaded by John Paul Magbitang (johnpaulmagbitang04@gmail.com)
lOMoARcPSD|15752181
 Fixed yearly allowance for entertainment 85,000
The fringe benefit tax is:
a. P50,400
b. P52,000
c. P84,000
d. P92,000
Gross Rent on Housing 129,200/95% = 136,000 x 50% = 68,000
Expense Account (CP & Pal Exp) 25,000 + 17500 42,500
Total 110,500
110,500/68% 162,500
162,500 x 32% 52,000
49. Continuing number 48, the income tax payable by Mistah is:
a. P13,200 old law
b. P27,760
c. P29,200
d. P43,360
Compensation(683,000 + 267,000) 950,000
Fixed Year Allowance 85,000
Personal Exemptions (50,000)
Taxable Income 985,000
Income Tax per Tabular (OLD LAW)
up to 500,000 125,000
Excess (485,000 x 32%) 155,200
Withholding Tax (267,000)
Income Tax Payable 13,200
Income Tax per Tabular (TRAIN LAW)
up to 800,000 130,000
Excess (185,000 x 30%) 55,500
Withholding Tax (267,000)
Income Tax Payable (Refund) (81,500)
Numbers 50 to 56 are based on the following information:
Domestic Export
Sales to private entities P500,000 P500,000
Sales to the government 500,000
Sales of exempt goods 500,000
Input taxes passed on by VAT-registered suppliers on:
Sales to private entities 30,000 20,000
Sales to the government 25,000
Sales of exempt goods 10,000
Purchase of depreciable capital goods not
attributable to any specific activity (monthly
amortization for 60 months) 120,000
Downloaded by John Paul Magbitang (johnpaulmagbitang04@gmail.com)
lOMoARcPSD|15752181
The sales to the government were subjected to the automatic deduction of the
1% creditable withholding tax (CWT) on its purchases from domestic
suppliers.
50. The value-added tax payable on the domestic sales to private
entities is:
a. Zero
b. (P10,000)
c. P60,000
d. P70,000
51. The total input taxes attributable to zero-rated sales is:
a. P20,000
b. P30,000
c. P50, 000
d. P60, 000
52. If the input taxes attributable to zero-rated are claimed as tax
credit, the net input value-added tax refundable is:
a. Zero
b. P40,000
c. P50,000
d. P60,000
53. The actual input taxes attributable to the domestic sales to the
government is:
a. P25,000
b. P30,000
c. P55,000
d. P65,000
54. The value-added tax payable on the domestic sales to the government
which was withheld as final withholding VAT is:
a. P5,000
b. P25,000
c. P30,000
d. P35,000
55. The journal entry to take up the domestic sales to the government
is:
a. Cash/AR 560,000
Sales 560,000
b. Cash/AR 560,000
Sales 500,000
Output tax 60,000
c. Cash/AR 535,000
Final withholding VAT 25,000
Sales 500,000
Output tax 60,000
Downloaded by John Paul Magbitang (johnpaulmagbitang04@gmail.com)
lOMoARcPSD|15752181
d. Cash/AR 530,000
Final withholding VAT 25,000
Creditable withholding tax 5,000
Sales 500,000
Output tax 60,000
Private Entities
VAT 0% VAT Government
OT 500,000 x 12% 60,000 60,000
500,000 x 0% -0-
IT (30,000) (20,000)
500,000 x 7% (35,000)
120,000 x 1 / 4 (30,000) (30,000)
VAT Payable -0-
Excess Input Tax (50,000)
FINAL VAT Withheld 25,000
56. The journal entry to reflect the excess of actual input VAT-over
the statutorily allowed input tax on the domestic sales to the
government is:
a. Output tax 60,000
Revenue and expense summary 20,000
Input tax 55,000
Final withholding VAT 25,000
b. Output tax 60,000
Input tax 15,000
Final withholding VAT 25,000
Revenue and expense summary 20,000
c. Output tax 60,000
Input tax 55,000
Revenue and expense summary 5,000
d. Output tax 60,000
Revenue and expense summary 20,000
Input tax 55,000
Cash/AP 25,000
57. Which one among the following statements is wrong?
a. Private Banks may be authorized to collect internal revenue taxes.
b. The Bureau of Customs (BOC) is also charged with the collection of
internal revenue taxes.
c. The local government unit’s (LGUs), such as cities, Municipalities
and Provinces, from part of the national tax system.
d. The bureau of Internal Revenue (BIR) is part of the administrative
machinery for the assessment and collection of internal revenue
taxes.
58. The prescriptive period for the issuance of a formal letter of
demand and final assessment notice (FLD/FAN) may not ordinarily be
stayed because of the lifeblood theory. There are certain
instances, however, where the running of the prescriptive period
Downloaded by John Paul Magbitang (johnpaulmagbitang04@gmail.com)
lOMoARcPSD|15752181
may be suspended. Which among the following instances is not among
the recognized exceptions which suspend the prescriptive period
within which to assess?
a. If the taxpayer is out of the country.
b. If the taxpayer changes his address, informing the commissioner of
such change.
c. Where the taxpayer request for and is granted a re investigation by
the Commissioner.
d. When the Commissioner of Internal Revenue (CIR) is prevented from
making an assessment and within 60 days thereafter.
59. An examination of a calendar year corporate taxpayer’s records
shows that it filed its final adjustment income tax return on
February 15, 2016 for its 2015 income. It subsequently filed an
amended income tax return March 21, 2016. Up to what date is the
Bureau of Internal Revenue (BIR) within which to issue a formal
letter of demand and final assessment notice (FLD/FAN)?
a. December 31, 2018 c. March 21, 2019
b. February 15, 2019 d. April 15, 2019
60. A closely held corporation has initially offered its shares in the
Philippine stock exchange (PSE). The following data pertain to the
initial public offering (IPO):
Number of shares sold in IPO 1,000,000 shares
Total outstanding shares before
the listing in the PSE 3,000,000 shares
Gross value in money of the IPO P20,000,000
The percentage tax due is:
a. P100,000 c. P400,000
b. P200,000 d. P800,000
Gross Value of IPO 20,000,000
OPT Rate 1M / 4M = 25% ( 4%, from 4%,2%, 1%) 4%
Percentage Tax 800,000
61. Shares of stock in a domestic corporation held as investment when
sold not through the local stock exchange shall be subject to:
a. 3% OPT or 12% VAT based on gross income
b. ½ of 1% based on gross selling price or gross value in money.
c. 5% on first P100, 000 capital gain; 10% on excess of P100, 000.
d. 4%;2%;1% based on gross selling price or gross value in money.
62. Malakas and Maganda were legally separated. They have six minor
children, all qualified to be claimed as additional exemptions for
income tax purposes. The court awarded custody of two of the
children to Malakas and three to Maganda, with Malakas directed
provide full financial support for them as well. The court awarded
the sixth child to Malakas’ parents with Malakas also providing
full financial support. Assuming that only Malakas is
gainfully earning while Maganda is not, for how many children could
Downloaded by John Paul Magbitang (johnpaulmagbitang04@gmail.com)
lOMoARcPSD|15752181
Malakas claim as additional exemptions when he files his annual
income tax return?
a. Two children c. Five Children
b. Three children d. Six children
63. Statement 1. Health and/or hospitalization insurance premium is
deductible from gross income by the spouse who claimed the
additional exemptions in case of married income taxpayers.
Statement 2. Health and/or hospitalization insurance premium paid
by an individual income taxpayer is deductible from gross income
for a minimum amount of P2, 400 provided the family’s gross income
for the year does not exceed P250, 000.
a. Both statements are true
b. Both statements are false
c. Statement 1 is true while statement 2 is false.
d. Statement 2 is true while statement 1 is false.
64. The following fringe benefits were given by an employer to its
employees for the quarter ending September 30, 2016:
De minimis benefits (not exceeding the maximum) P 200,000
Reimbursed expense of rank and file employees 400,000
Housing benefits to managers and supervisors
(Representing total rents) 680,000
The fringe benefit tax payable for the quarter is:
a. P160, 000 c. P442, 353
b. P320, 000 d. P502, 353
Housing Benefits to Manager
680,000 x 50% = 340,000 / 68% x 32% = 160,000
65. Continuing number 64, how much would be the total deductions from
gross income which may be claimed by the employer?
a. P1,100,000 c. P1,440,000
b. P1,280,000 d. P1,600,000
De minimis benefits (not exceeding the maximum) P 200,000
Reimbursed expense of rank and file employees 400,000
Housing benefits to managers and supervisors
(Representing total rents) 680,000
Fringe Benefit Tax Expense 160,000
Total Deductions from Gross Income 1,440,000
66. Going back to the number 64, the employer shall file a remittance
return of the final tax on fringe benefit and pay the tax withheld
within:
a. 5 days from the close of each month.
b. 10 days from the close of each month.
c. 10 days from the close of each fiscal quarter.
d. 10 days from the close of each calendar quarter.
Downloaded by John Paul Magbitang (johnpaulmagbitang04@gmail.com)
lOMoARcPSD|15752181
67. Kalansay, a native of Negros, died leaving a property acquired by
purchase from Naty Gok who died 3 ½ years ago. The property is now
a Kalansay’s gross estate. The estate’s vanishing deduction rate
is:
a. 0%
b. 20%
c. 40%
d. 60%
Property is not gratuitously acquired by the present Estate
68. A lessor of residential units has the following gross receipts for
2016:
Monthly Number
Rent per unit of units Total
P 9,000 5 P 540,000
10,000 5 600,000
11,000 5 660,000
12,000 5 720,000
The output VAT is:
a. P0 c. P270,000
b. P230,340 d. P302,400
Leasing on a residential units is VAT Exempt
69. Using the same facts in number 68, only that the lessor is a VAT-
registered person, the OPT tax due is:
a. P0 c. P67,500
b. P57,585 d. P75,600
OPT Exempt also
70. The deduction allowed for the payment of premium on hospitalization
insurance during the taxable year by a resident citizen amounting
to P3, 000 for the months of August to December is:
a. P 3,000
b. P2, 400
c. P1, 200
d. P1, 000
200 x 5 months = 1,000
71. The following are among the constitutional limitation on the power
of taxation. Which one is not?
a. Due process clause
b. Non-impairment clause
c. Equal protection clause
d. No imprisonment for non-payment of tax
Downloaded by John Paul Magbitang (johnpaulmagbitang04@gmail.com)
lOMoARcPSD|15752181
72. Pacmom filed her income tax from return for 2015 on May 16, 2016
and paid the tax of P50,000. Upon audit by the BIR an assessment
notice was issued on April 30, 2019, requiring Pacmom to pay a
deficiency tax of P 75,000 not later than July 30, 2019. Pacmom
will:
a. False prescription as defense
b. Request for an extension of time to pay the deficiency income tax.
c. Go to the court of tax appeals to appeal the assessment made by the
BIR.
d. Ignore the assessment as the date of collection is already way
beyond three years, covering taxable year 2015.
73. Date assessment was received March 08, 2016
Date petition for reinvestigation
was filed by the BIR March 18, 2016
Date of filing of documents to
support the petition April 08, 2016
Date decision of denial of the
petition was received April 28, 2016
The last day to appeal to the CTA is on:
a. April 17, 2016 c. May 8, 2016
b. April 17, 2016 d. May 28, 2016
74. Using the same facts in number 73, only that instead of going to
the CTA, a request for reconsideration was filed with the BIR on
May 7, 2016. Date decision of denial of the request for
reconsideration was received on June 2, 2016. The last day to
appeal to the CTA is on:
a. June 7, 2016 c. June 23, 2016
b. June 21, 2016 d. July 2, 2016
75. Continuing number 74 only that instead of receiving a decision
denying a request for reconsideration, a revised assessment was
received on June 2, 2016. The last day to appeal to the CTA is on:
a. June 7, 2016 c. June 23, 2016
b. June 21, 2016 d. July 2, 2016
30 days from June 2, 2016
76. Statement 1. The Bureau of Internal Revenue (BIR) has as its powers
and duties the assessment and collection of the national internal
revenue taxes, fees, and changes and the enforcement of all
fortitudes, penalties, and fines connected therewith.
Statement 2. The Bureau of Customs (BOC) has its powers and duties
the assessment and collection of awful revenues from imported and
exported articles and all other dues, fees, charges, fines, and
penalties accruing under the tariff and customs laws.
a. Both statements are correct.
b. Both statements are wrong.
c. Statement 1 is correct but statement 2 is wrong.
d. Statement 2 is correct but statement 1 is wrong.
Downloaded by John Paul Magbitang (johnpaulmagbitang04@gmail.com)
lOMoARcPSD|15752181
77. A customs duty that imposes both advance and specific customs
duties on imported or exported articles is:
a. Anti-dumping duty.
b. Countervailing duty.
c. Discriminating duty.
d. Compound customs duty.
78. Consider the following statements:
i. The imposition of customs duties also assists in economic
development.
ii. Customs duties are sometimes imposed to protect local
consumers.
iii. The purpose of regular customs duties is to raise
revenues to meet the needs of government.
iv. Compound customs duties are computed only on the basis of
units of measure, such as weight, measurement, quantity,
etc.
a. All the above statement is correct.
b. Only statements I, II and III are correct.
c. Only statements I, II and IV are correct.
d. Only statement I, III and IV are correct.
Numbers 79 to 80 are based on the following information pertaining to
taxable year 2016:
Phil. USA
Sales P500,000 P600,000
Sales discounts,
Returns and allowances 20,000 50,000
Cost of sales 230,000 250,000
Rent expenses 50,000 80,000
Salaries and wages 50,000 60,000
Interest expenses 10,000 -
Entertainment and presentation 10,000 -
Contribution to charitable institution 10,000 -
Interest on bank deposit 10,000 20,000
Royalty- musical compositions 20,000 20,000
Dividends from domestic corporation 10,000 -
79. The taxable income if the taxpayer is a domestic corporation (DC)
and resident citizen (RC), under:
Itemized Deductions Optional Deduction
DC RC DC RC
a. P320,000 P270,000 P330,000 P280,000
b. P328,150 P278,150 P354,000 P592,000
c. P280,000 P230,000 P378,000 P328,000
d. P288,150 P238,150 P330,000 P568,000
ITEMIZED DEDUCTIONS DC RC
Passive Income 40,000 40,000
Sales 1,100,000 1,100,000
Downloaded by John Paul Magbitang (johnpaulmagbitang04@gmail.com)
lOMoARcPSD|15752181
Sales Disc, Ret & Allowances (70,000) (70,000)____
Net Sales 1,030,000 1,030,000
Cost of Good Sold (480,000) (480,000)
Business Gross Income 550,000 550,000
Total Gross Income 590,000 590,000
Other Expenses (240,000) (240,000)
Interest 10,000 - (10,000 x 33%) (6,700) (6,700)
Entert & Repre (1,030,000 x .5%) (5,150) (5,150)
Net Income Before CC 338,150 338,150
CC Actual < Limit (10,000) (10,000)
Personal Exemption -0- (50,000)
Taxable Income 328,000 278,150
OPTIONAL DEDUCTION DC RC
Passive Income 40,000 40,000
Net Sales 1,030,000
Business Gross Income 550,000
Total Gross Income/
Gross Receipt & Net Sales 590,000 1,070,000
40% OSD (236,000) (428,000)
Personal Exemption -0- (50,000)
Taxable Income 354,000 592,000
80. The taxable income if the taxpayer is a resident foreign
corporation (RFC) and non-resident citizen (NRC):
RFC NRC
a. P160,000 P270,000
b. P130,900 P 80,900
c. P133,855 P 80,900
d. P120,000 P 70,000
ITEMIZED DEDUCTIONS RFC NRC
Passive Income
Sales 500,000 500,000
Sales Disc, Ret & Allowances (20,000) (20,000) _
Net Sales 480,000 480,000
Cost of Good Sold (230,000) (230,000)
Other Expenses (100,000) (100,000)
Interest 10,000 - (10,000 x 33%) (6,700) (6,700)
Entert & Repre (1,030,000 x .5%) (2,400) (2,400)
Net Income Before CC 140,900 140,900
CC 140,900 x 5% (7,045) (10,000)
Personal Exemption -0- (50,000)
Taxable Income 133,855 80,900
Downloaded by John Paul Magbitang (johnpaulmagbitang04@gmail.com)
lOMoARcPSD|15752181
TAXATION QUIZZER PART 3
1. The following are common to the inherent power of taxation, power of
eminent domain and police power, except for which of the following?
a. They are necessary attributes to the sovereignty.
b. They interfere with private rights and property.
c. They affect all persons or the public.
d. They are legislative in implementation.
2. In case of ambiguity, tax laws shall be interpreted:
a. Strictly against the taxpayer.
b. Liberally against the government.
c. Liberally in favor of the taxpayer.
d. Liberally in favor of the government.
3. Which of the following statements is not correct?
a. Taxes may be imposed to raise revenue or to provide incentives
or disincentives for certain activities within the state.
b. The state can have the power of taxation even if the
constitution does not expressly give it the power to tax.
c. For the exercise of the power of taxation, the state can tax
anything at anytime.
d. The provisions of taxation in the Philippine Constitution are
grants of power and not limitation on the taxing power.
4. Which of the following is not a direct tax?
a. Immigration tax c. Income tax
b. Transfer tax d. Contractor’s tax (Now VAT)
5. In which situation will a CPA’s signature be necessary?
a. When the value of the gross estate is P2,000,000 and above.
b. When the value of the gross estate exceeds P2,000,000.
c. Regardless of the value where the gross estate consists or
registered or registrable property.
d. When the value of the gross estate exceeds P200,000, although
exempt from transfer tax.
6. A seller of goods is non-VAT registered. His annual gross sales amount
to P1,919,500. To what business tax is he liable?
a. 3% tax on VAT-exempt persons c. 3% common carrier’s tax
b. 12% value-added tax d. Not subject to any percentage tax
7. Abigail sold through the local stock exchange, 10,000 PLDT shares that
she bought 2 years ago. Abigail sold the shares for P2 million and
realized a net gain of P200,000. The transactions is
a. Subject to regular income tax rates for individuals but only 50%
shall be recognized because it is a long-term capital gain.
b. Subject to capital gains tax amounting to P15,000.
c. Subject to percentage tax amounting to P10,000.
d. Subject to regular income tax rates for individual under Section
24 (A).
Downloaded by John Paul Magbitang (johnpaulmagbitang04@gmail.com)
lOMoARcPSD|15752181
8. Proceeds of life insurance to the extent of the amount receivable by
the estate of the deceased, his executor or administrator under
policies taken out by the dependent upon his own life shall be
I. Part of the gross estate irrespective of whether or
not the insured retained the power of revocation.
II. Not part of the gross estate if the beneficiary is
irrevocable.
III. Part of the gross income if the designation of the
beneficiary is revocable.
IV. Not part of the gross income irrespective of whether
or not the insured retained the power of revocation.
a. I and II
b. I and III
c. I and IV
d. only I
9. One of the following is not correct
Deduction Maximum
a. Funeral Expenses P200,000
b. Family home P2,000,000
c. Medical expenses P500,000
d. Standard Deduction P1,000,000
10. One of them is not considered non-resident citizen?
a. A citizen of the Philippines who establishes to the satisfaction
of the Commissioner the fact of his physical presence abroad
with a definite intention to reside therein.
b. A citizen of the Philippines who leaves the Philippines during
the taxable year to reside abroad, either as an immigrant or for
employment on permanent basis.
c. A citizen of the Philippines who works and derives income from
abroad and whose employment thereat requires him to be
physically present abroad most of the time during the taxable
year.
d. A citizen of the Philippines who went on a business trip abroad
and stayed therein most of the time during the year.
11. Z is a Filipino immigrant living in the United States for more than 10
years. He is retired and he came back to the Philippines as a
balikbayan. Every time he comes to the Philippines, he stays here for
about a month. He regularly receives a pension from his former
employer in the United States, amounting to USD1,000 a month. While in
the Philippines, with his pension pay from his former employer, he
purchased 3 condominium units in Makati, which he is renting out for
P15,000 a month each. Does the USD1,000 pension become taxable because
he is now in the Philippines?
a. Yes. Income received in the Philippines by non-resident citizens
is taxable.
b. Yes. Income received in the Philippines or abroad by non-
resident citizens is taxable.
Downloaded by John Paul Magbitang (johnpaulmagbitang04@gmail.com)
lOMoARcPSD|15752181
c. No. Income earned abroad by non-resident citizens is no longer
taxable in the Philippines.
d. No, the pension is exempt from taxation being one of the
exclusions from gross income.
12. Which of the following income earners is required to file income tax
return?
a. Minimum wage earners
b. Non-resident alien not engaged in business
c. An individual with respect to pure compensation income deriving
from such sources within the Philippines, the income tax on
which has been correctly withheld and that an individual
deriving compensation from one employer at any time during the
taxable year
d. General professional partnership
13. Which of the following fringe benefit is not subject to fringe benefit
tax?
a. a. Housing benefit c. De minimis benefit
b. b. Expense account d. Vehicle benefit
14. In the case of a taxpayer, only the following percentages of the gain
or loss recognized upon the sale or exchange of a capital asset shall
be taken into account in computing net capital gain, net capital loss
and net income.
I. Statement 1 One hundred percent (100%) if the capital asset
has been held for not more than twelve (12) months by a
taxpayer.
II. Statement 2 Fifty percent (50%) if the capital asset has been
held for more than twelve (12) months by a taxpayer.
a. True, true c. False, false
b. True, false d. False, true
15. 15.A, worked for a manufacturing firm but due to business reverses,
the firm offered a voluntary redundancy program in order to reduce
overhead expenses. Under the program, an employee who offered to
resign would be given separation pay equivalent to his 3 months basic
salary for every year of service. A accepted the offer and received
P800,000 as separation pay under the program. After all employees who
accepted the offer were paid, the firm found its overhead still
excessive. Hence, it adopted another program, where various
unprofitable departments were closed. As a result, B was separated
from the service. B also received P800,000 as separation pay. At the
time of separation both A and B have rendered at least 10 years of
service but A was 55 years old while B was only 45 years old. As a
result,
a. Both amounts are exempt from income tax
b. Both amounts are subject to income tax
c. Only Mr. A is subject to income tax
d. Only Mr. B is subject to income tax
Downloaded by John Paul Magbitang (johnpaulmagbitang04@gmail.com)
lOMoARcPSD|15752181
16. The following shall not be subject to estate tax
a. The merger of usufruct in the owner of naked title
b. The transmission or delivery of inheritance or legacy by the
fiduciary heir or legatee to fideicommissary
c. The transmission from the first heir, legatee, or donee in favor
of another beneficiary in accordance with the desire of the
predecessor
d. All bequest, devises, legacies or transfers in social welfare,
cultural and charitable institutions
17. Which of the following transactions is subject to zero-rated value-
added tax?
a. Services rendered to persons engaged in international shipping
or air transport operations
b. Services rendered banks, non-bank financial intermediaries
c. Generation, transmission and distribution of electricity
d. Services rendered by professionals such as CPA, Physicians, and
Lawyers.
18. Which of the following businesses is allowed presumptive input value-
added tax?
a. Manufacturer of canned goods
b. Manufacturer of packed juices
c. Manufacturer of packed noodles
d. Manufacturer of dried fish
19. Which of the following shall be allowed as a deduction from the gross
income?
a. Depreciation for vehicle for land transport used for personal
purposes the value of which does not exceed P2,400,000.
b. Depreciation for vehicle for land transport used in the business
the value of which exceeds P2,400,000.
c. Depreciation for land vehicles the value of which exceeds
P2,400,000 where the taxpayer’s main line of business is
transport operations or lease of transportation equipment and
the vehicles purchased are used in said operations.
d. Depreciation for airplanes and/or aircraft the value of which
exceeds P2,400,000 where the taxpayer’s main line of business is
sale of goods or properties and the vehicles purchased are used
in said operations.
20. A, at the time of retirement, had 1,000 pieces of merchandise which
was deemed sold at a value of P20,000, with an output tax of P2,400.
After retirement, A sold to B 500 pieces of these for P12,000. In the
contract of sale or invoice, A stated sales invoice number wherein the
output tax on deemed sale was imposed and the corresponding tax paid
on the 500 pieces. He prepared the following invoice:
I. Gross selling price P10,800
II. VAT previously paid on deemed sale 1,200
III. Total P12,000
Downloaded by John Paul Magbitang (johnpaulmagbitang04@gmail.com)
lOMoARcPSD|15752181
quiz-questions-and-answers.pdf
quiz-questions-and-answers.pdf
quiz-questions-and-answers.pdf
quiz-questions-and-answers.pdf
quiz-questions-and-answers.pdf
quiz-questions-and-answers.pdf
quiz-questions-and-answers.pdf
quiz-questions-and-answers.pdf
quiz-questions-and-answers.pdf
quiz-questions-and-answers.pdf

More Related Content

What's hot

01 Chapter 1 and 2 Taxation 2
01 Chapter 1 and 2 Taxation 2 01 Chapter 1 and 2 Taxation 2
01 Chapter 1 and 2 Taxation 2
Flab Villasencio
 
03 chapter 4 deductions from gross estate part 02
03 chapter 4 deductions from gross estate part 0203 chapter 4 deductions from gross estate part 02
03 chapter 4 deductions from gross estate part 02
Flab Villasencio
 
Module 3 law of contracts
Module 3  law of contractsModule 3  law of contracts
Module 3 law of contracts
Gichelle Amon
 
Value Added Tax (Taxable Sales) Philippines
Value Added Tax (Taxable Sales) PhilippinesValue Added Tax (Taxable Sales) Philippines
Value Added Tax (Taxable Sales) Philippines
Karla J. Medina
 

What's hot (20)

01 Chapter 1 and 2 Taxation 2
01 Chapter 1 and 2 Taxation 2 01 Chapter 1 and 2 Taxation 2
01 Chapter 1 and 2 Taxation 2
 
Pledge and Chattel Mortgage
Pledge and Chattel MortgagePledge and Chattel Mortgage
Pledge and Chattel Mortgage
 
Contracts
ContractsContracts
Contracts
 
Law On Obligations and Contracts (midterm exam)
Law On Obligations and Contracts (midterm exam)Law On Obligations and Contracts (midterm exam)
Law On Obligations and Contracts (midterm exam)
 
03 chapter 4 deductions from gross estate part 02
03 chapter 4 deductions from gross estate part 0203 chapter 4 deductions from gross estate part 02
03 chapter 4 deductions from gross estate part 02
 
Module 3 law of contracts
Module 3  law of contractsModule 3  law of contracts
Module 3 law of contracts
 
Value Added Tax (Taxable Sales) Philippines
Value Added Tax (Taxable Sales) PhilippinesValue Added Tax (Taxable Sales) Philippines
Value Added Tax (Taxable Sales) Philippines
 
Inherent Powers of the State
Inherent Powers of the StateInherent Powers of the State
Inherent Powers of the State
 
Sales finals reviewer
Sales finals reviewerSales finals reviewer
Sales finals reviewer
 
187153232 obligations-and-contracts
187153232 obligations-and-contracts187153232 obligations-and-contracts
187153232 obligations-and-contracts
 
Public finance for BSBA
Public finance for BSBAPublic finance for BSBA
Public finance for BSBA
 
Allocation and-Distribution-of-net-surplus
Allocation and-Distribution-of-net-surplusAllocation and-Distribution-of-net-surplus
Allocation and-Distribution-of-net-surplus
 
Law on Obligations and Contracts
Law on Obligations and ContractsLaw on Obligations and Contracts
Law on Obligations and Contracts
 
Partnership
PartnershipPartnership
Partnership
 
Blt 134 chapter 2
Blt 134 chapter 2Blt 134 chapter 2
Blt 134 chapter 2
 
IA2 2 Premium
IA2 2 PremiumIA2 2 Premium
IA2 2 Premium
 
PARTNERSHIP dissolution LUPISAN
PARTNERSHIP dissolution LUPISANPARTNERSHIP dissolution LUPISAN
PARTNERSHIP dissolution LUPISAN
 
Taxing powers, scope and limitations of nga and lgu
Taxing powers, scope and limitations of nga and lguTaxing powers, scope and limitations of nga and lgu
Taxing powers, scope and limitations of nga and lgu
 
Law on obligations and contracts
Law on obligations and contractsLaw on obligations and contracts
Law on obligations and contracts
 
BUSLAW1: Sales Topic 1
BUSLAW1: Sales Topic 1BUSLAW1: Sales Topic 1
BUSLAW1: Sales Topic 1
 

Similar to quiz-questions-and-answers.pdf

26 Answers to knowledge and understanding questions.pdf
26 Answers to knowledge and understanding questions.pdf26 Answers to knowledge and understanding questions.pdf
26 Answers to knowledge and understanding questions.pdf
alenradak
 
Multiple choice Tax QuestionsPeter and Eileen are marrie.docx
Multiple choice Tax QuestionsPeter and Eileen are marrie.docxMultiple choice Tax QuestionsPeter and Eileen are marrie.docx
Multiple choice Tax QuestionsPeter and Eileen are marrie.docx
adelaidefarmer322
 
Peter and Eileen are married and live in a common law state. Peter.docx
Peter and Eileen are married and live in a common law state. Peter.docxPeter and Eileen are married and live in a common law state. Peter.docx
Peter and Eileen are married and live in a common law state. Peter.docx
herbertwilson5999
 
Legal InfluencesDo we have too many laws Does the existence o.docx
Legal InfluencesDo we have too many laws Does the existence o.docxLegal InfluencesDo we have too many laws Does the existence o.docx
Legal InfluencesDo we have too many laws Does the existence o.docx
SHIVA101531
 
Finc 430Quiz 1 will be posted on Friday the 13th, but will need.docx
Finc 430Quiz 1 will be posted on Friday the 13th, but will need.docxFinc 430Quiz 1 will be posted on Friday the 13th, but will need.docx
Finc 430Quiz 1 will be posted on Friday the 13th, but will need.docx
voversbyobersby
 
Income Tax 1. Which of the following can be deducted as an itemiz.docx
Income Tax  1. Which of the following can be deducted as an itemiz.docxIncome Tax  1. Which of the following can be deducted as an itemiz.docx
Income Tax 1. Which of the following can be deducted as an itemiz.docx
doylymaura
 
Adoption Tax Credit: A Guide for Advocates
Adoption Tax Credit: A Guide for AdvocatesAdoption Tax Credit: A Guide for Advocates
Adoption Tax Credit: A Guide for Advocates
Madeline Daniels
 
Page 1 of 6 HA3042 Taxation Law Individual Assignment T1..docx
Page 1 of 6 HA3042 Taxation Law Individual Assignment T1..docxPage 1 of 6 HA3042 Taxation Law Individual Assignment T1..docx
Page 1 of 6 HA3042 Taxation Law Individual Assignment T1..docx
AASTHA76
 

Similar to quiz-questions-and-answers.pdf (20)

toaz-tax-multiple-choice.pdf
toaz-tax-multiple-choice.pdftoaz-tax-multiple-choice.pdf
toaz-tax-multiple-choice.pdf
 
Inheritance Tax Seminar By Zee Shan Smartfield Accountants In Leicester
Inheritance Tax Seminar By Zee Shan Smartfield Accountants In LeicesterInheritance Tax Seminar By Zee Shan Smartfield Accountants In Leicester
Inheritance Tax Seminar By Zee Shan Smartfield Accountants In Leicester
 
26 Answers to knowledge and understanding questions.pdf
26 Answers to knowledge and understanding questions.pdf26 Answers to knowledge and understanding questions.pdf
26 Answers to knowledge and understanding questions.pdf
 
Wassim Zhani Federal Taxation Chapter 1 An Overview of Federal Taxation.pdf
Wassim Zhani Federal Taxation Chapter 1 An Overview of Federal Taxation.pdfWassim Zhani Federal Taxation Chapter 1 An Overview of Federal Taxation.pdf
Wassim Zhani Federal Taxation Chapter 1 An Overview of Federal Taxation.pdf
 
Multiple choice Tax QuestionsPeter and Eileen are marrie.docx
Multiple choice Tax QuestionsPeter and Eileen are marrie.docxMultiple choice Tax QuestionsPeter and Eileen are marrie.docx
Multiple choice Tax QuestionsPeter and Eileen are marrie.docx
 
Peter and Eileen are married and live in a common law state. Peter.docx
Peter and Eileen are married and live in a common law state. Peter.docxPeter and Eileen are married and live in a common law state. Peter.docx
Peter and Eileen are married and live in a common law state. Peter.docx
 
Legal InfluencesDo we have too many laws Does the existence o.docx
Legal InfluencesDo we have too many laws Does the existence o.docxLegal InfluencesDo we have too many laws Does the existence o.docx
Legal InfluencesDo we have too many laws Does the existence o.docx
 
Sample for Solution Manual Income Tax Fundamentals 2024 42th Edition by Whitt...
Sample for Solution Manual Income Tax Fundamentals 2024 42th Edition by Whitt...Sample for Solution Manual Income Tax Fundamentals 2024 42th Edition by Whitt...
Sample for Solution Manual Income Tax Fundamentals 2024 42th Edition by Whitt...
 
Finc 430Quiz 1 will be posted on Friday the 13th, but will need.docx
Finc 430Quiz 1 will be posted on Friday the 13th, but will need.docxFinc 430Quiz 1 will be posted on Friday the 13th, but will need.docx
Finc 430Quiz 1 will be posted on Friday the 13th, but will need.docx
 
Wassim Zhani Chapter 16 Family Tax Planning.pdf
Wassim Zhani Chapter 16 Family Tax Planning.pdfWassim Zhani Chapter 16 Family Tax Planning.pdf
Wassim Zhani Chapter 16 Family Tax Planning.pdf
 
Reverse percentage, profit, loss and discount
Reverse percentage, profit, loss and discountReverse percentage, profit, loss and discount
Reverse percentage, profit, loss and discount
 
Fin quiz
Fin quizFin quiz
Fin quiz
 
Income Tax 1. Which of the following can be deducted as an itemiz.docx
Income Tax  1. Which of the following can be deducted as an itemiz.docxIncome Tax  1. Which of the following can be deducted as an itemiz.docx
Income Tax 1. Which of the following can be deducted as an itemiz.docx
 
Gift tax presentation
Gift tax presentationGift tax presentation
Gift tax presentation
 
Adoption Tax Credit: A Guide for Advocates
Adoption Tax Credit: A Guide for AdvocatesAdoption Tax Credit: A Guide for Advocates
Adoption Tax Credit: A Guide for Advocates
 
Page 1 of 6 HA3042 Taxation Law Individual Assignment T1..docx
Page 1 of 6 HA3042 Taxation Law Individual Assignment T1..docxPage 1 of 6 HA3042 Taxation Law Individual Assignment T1..docx
Page 1 of 6 HA3042 Taxation Law Individual Assignment T1..docx
 
Wassim Zhani Chapter 15 Income Taxation of Estates and Trusts.pdf
Wassim Zhani Chapter 15 Income Taxation of Estates and Trusts.pdfWassim Zhani Chapter 15 Income Taxation of Estates and Trusts.pdf
Wassim Zhani Chapter 15 Income Taxation of Estates and Trusts.pdf
 
Tax relief ya 2021
Tax relief ya 2021Tax relief ya 2021
Tax relief ya 2021
 
Webinar: How Can Housing Associations Respond To Another Round Of Welfare Ref...
Webinar: How Can Housing Associations Respond To Another Round Of Welfare Ref...Webinar: How Can Housing Associations Respond To Another Round Of Welfare Ref...
Webinar: How Can Housing Associations Respond To Another Round Of Welfare Ref...
 
Rental Income Accounting Seminar Delivered By Smartfield Accountants in Leice...
Rental Income Accounting Seminar Delivered By Smartfield Accountants in Leice...Rental Income Accounting Seminar Delivered By Smartfield Accountants in Leice...
Rental Income Accounting Seminar Delivered By Smartfield Accountants in Leice...
 

Recently uploaded

Challenges and Opportunities: A Qualitative Study on Tax Compliance in Pakistan
Challenges and Opportunities: A Qualitative Study on Tax Compliance in PakistanChallenges and Opportunities: A Qualitative Study on Tax Compliance in Pakistan
Challenges and Opportunities: A Qualitative Study on Tax Compliance in Pakistan
vineshkumarsajnani12
 
Jual obat aborsi Hongkong ( 085657271886 ) Cytote pil telat bulan penggugur k...
Jual obat aborsi Hongkong ( 085657271886 ) Cytote pil telat bulan penggugur k...Jual obat aborsi Hongkong ( 085657271886 ) Cytote pil telat bulan penggugur k...
Jual obat aborsi Hongkong ( 085657271886 ) Cytote pil telat bulan penggugur k...
Klinik kandungan
 
Goa Call Girls Just Call 👉📞90042XXXX7 Top Class Call Girl Service Available
Goa Call Girls Just Call 👉📞90042XXXX7 Top Class Call Girl Service AvailableGoa Call Girls Just Call 👉📞90042XXXX7 Top Class Call Girl Service Available
Goa Call Girls Just Call 👉📞90042XXXX7 Top Class Call Girl Service Available
Call Girls Mumbai
 

Recently uploaded (20)

Challenges and Opportunities: A Qualitative Study on Tax Compliance in Pakistan
Challenges and Opportunities: A Qualitative Study on Tax Compliance in PakistanChallenges and Opportunities: A Qualitative Study on Tax Compliance in Pakistan
Challenges and Opportunities: A Qualitative Study on Tax Compliance in Pakistan
 
JIND CALL GIRL ❤ 8272964427❤ CALL GIRLS IN JIND ESCORTS SERVICE PROVIDE
JIND CALL GIRL ❤ 8272964427❤ CALL GIRLS IN JIND ESCORTS SERVICE PROVIDEJIND CALL GIRL ❤ 8272964427❤ CALL GIRLS IN JIND ESCORTS SERVICE PROVIDE
JIND CALL GIRL ❤ 8272964427❤ CALL GIRLS IN JIND ESCORTS SERVICE PROVIDE
 
PALWAL CALL GIRL ❤ 82729*64427❤ CALL GIRLS IN PALWAL ESCORTS
PALWAL CALL GIRL ❤ 82729*64427❤ CALL GIRLS IN PALWAL ESCORTSPALWAL CALL GIRL ❤ 82729*64427❤ CALL GIRLS IN PALWAL ESCORTS
PALWAL CALL GIRL ❤ 82729*64427❤ CALL GIRLS IN PALWAL ESCORTS
 
The Art of Decision-Making: Navigating Complexity and Uncertainty
The Art of Decision-Making: Navigating Complexity and UncertaintyThe Art of Decision-Making: Navigating Complexity and Uncertainty
The Art of Decision-Making: Navigating Complexity and Uncertainty
 
QSM Chap 10 Service Culture in Tourism and Hospitality Industry.pptx
QSM Chap 10 Service Culture in Tourism and Hospitality Industry.pptxQSM Chap 10 Service Culture in Tourism and Hospitality Industry.pptx
QSM Chap 10 Service Culture in Tourism and Hospitality Industry.pptx
 
A DAY IN THE LIFE OF A SALESPERSON .pptx
A DAY IN THE LIFE OF A SALESPERSON .pptxA DAY IN THE LIFE OF A SALESPERSON .pptx
A DAY IN THE LIFE OF A SALESPERSON .pptx
 
10 Influential Leaders Defining the Future of Digital Banking in 2024.pdf
10 Influential Leaders Defining the Future of Digital Banking in 2024.pdf10 Influential Leaders Defining the Future of Digital Banking in 2024.pdf
10 Influential Leaders Defining the Future of Digital Banking in 2024.pdf
 
Bhubaneswar Call Girl Just Call♥️ 8084732287 ♥️Top Class Call Girl Service Av...
Bhubaneswar Call Girl Just Call♥️ 8084732287 ♥️Top Class Call Girl Service Av...Bhubaneswar Call Girl Just Call♥️ 8084732287 ♥️Top Class Call Girl Service Av...
Bhubaneswar Call Girl Just Call♥️ 8084732287 ♥️Top Class Call Girl Service Av...
 
Goal Presentation_NEW EMPLOYEE_NETAPS FOUNDATION.pptx
Goal Presentation_NEW EMPLOYEE_NETAPS FOUNDATION.pptxGoal Presentation_NEW EMPLOYEE_NETAPS FOUNDATION.pptx
Goal Presentation_NEW EMPLOYEE_NETAPS FOUNDATION.pptx
 
Unveiling Falcon Invoice Discounting: Leading the Way as India's Premier Bill...
Unveiling Falcon Invoice Discounting: Leading the Way as India's Premier Bill...Unveiling Falcon Invoice Discounting: Leading the Way as India's Premier Bill...
Unveiling Falcon Invoice Discounting: Leading the Way as India's Premier Bill...
 
Ital Liptz - all about Itai Liptz. news.
Ital Liptz - all about Itai Liptz. news.Ital Liptz - all about Itai Liptz. news.
Ital Liptz - all about Itai Liptz. news.
 
Jual obat aborsi Hongkong ( 085657271886 ) Cytote pil telat bulan penggugur k...
Jual obat aborsi Hongkong ( 085657271886 ) Cytote pil telat bulan penggugur k...Jual obat aborsi Hongkong ( 085657271886 ) Cytote pil telat bulan penggugur k...
Jual obat aborsi Hongkong ( 085657271886 ) Cytote pil telat bulan penggugur k...
 
Arti Languages Pre Seed Teaser Deck 2024.pdf
Arti Languages Pre Seed Teaser Deck 2024.pdfArti Languages Pre Seed Teaser Deck 2024.pdf
Arti Languages Pre Seed Teaser Deck 2024.pdf
 
PHX May 2024 Corporate Presentation Final
PHX May 2024 Corporate Presentation FinalPHX May 2024 Corporate Presentation Final
PHX May 2024 Corporate Presentation Final
 
Goa Call Girls Just Call 👉📞90042XXXX7 Top Class Call Girl Service Available
Goa Call Girls Just Call 👉📞90042XXXX7 Top Class Call Girl Service AvailableGoa Call Girls Just Call 👉📞90042XXXX7 Top Class Call Girl Service Available
Goa Call Girls Just Call 👉📞90042XXXX7 Top Class Call Girl Service Available
 
Buy gmail accounts.pdf buy Old Gmail Accounts
Buy gmail accounts.pdf buy Old Gmail AccountsBuy gmail accounts.pdf buy Old Gmail Accounts
Buy gmail accounts.pdf buy Old Gmail Accounts
 
Falcon Invoice Discounting: Empowering Your Business Growth
Falcon Invoice Discounting: Empowering Your Business GrowthFalcon Invoice Discounting: Empowering Your Business Growth
Falcon Invoice Discounting: Empowering Your Business Growth
 
Thompson_Taylor_MBBS_PB1_2024-03 (1)- Project & Portfolio 2.pptx
Thompson_Taylor_MBBS_PB1_2024-03 (1)- Project & Portfolio 2.pptxThompson_Taylor_MBBS_PB1_2024-03 (1)- Project & Portfolio 2.pptx
Thompson_Taylor_MBBS_PB1_2024-03 (1)- Project & Portfolio 2.pptx
 
Lundin Gold - Q1 2024 Conference Call Presentation (Revised)
Lundin Gold - Q1 2024 Conference Call Presentation (Revised)Lundin Gold - Q1 2024 Conference Call Presentation (Revised)
Lundin Gold - Q1 2024 Conference Call Presentation (Revised)
 
Falcon Invoice Discounting: Unlock Your Business Potential
Falcon Invoice Discounting: Unlock Your Business PotentialFalcon Invoice Discounting: Unlock Your Business Potential
Falcon Invoice Discounting: Unlock Your Business Potential
 

quiz-questions-and-answers.pdf

  • 1. StuDocu is not sponsored or endorsed by any college or university QUIZ, questions and answers Bachelor of Science Nursing (La Consolacion University Philippines) StuDocu is not sponsored or endorsed by any college or university QUIZ, questions and answers Bachelor of Science Nursing (La Consolacion University Philippines) Downloaded by John Paul Magbitang (johnpaulmagbitang04@gmail.com) lOMoARcPSD|15752181
  • 2. TAXATION QUIZZER PART 1 BASIC PRINCIPLES OF TAXATION 1. Which theory in taxation states that without taxes, a government would be paralyzed for lack of power to activate and operate it, resulting in its destruction? a. Power to destroy theory b. Lifeblood theory c. Sumptuary theory d. Symbiotic doctrine 2. The actual effort exerted by the government to effect the exaction of what is due from the taxpayer is known as a. Assessment. b. Levy. c. Payment. d. Collection. 3. Although the power of taxation is basically legislative in character, it is NOT the function of Congress to a. Fix with certainty the amount of taxes. b. Collect the tax levied under the law. c. Identify who should collect the tax. d. Determine who should be subject to the tax. 4. An example of a tax where the concept of progressivity finds application is the a. Income tax on individuals. b. Excise tax on petroleum products. c. Value-added tax on certain articles. d. Amusement tax on boxing exhibitions. 5. Ligaya Educational Foundation, Inc., a stock educational institution organized for profit, decided to lease for commercial use a 1,500 sq. m. portion of its school. The school actually, directly, and exclusively use the rents for the maintenance of its school buildings, including payment of janitorial services. Is the leased portion subject to real property tax? a. Yes, since Lualhati is a stock and for profit educational institution. b. No, since the school actually, directly, and exclusively used the rents for educational purposes. c. No, but it may be subject to income taxation on the rents it receives. d. Yes, since the leased portion is not actually, directly, and exclusively used for educational purposes. 6. Which among the following concepts of taxation is the basis for the situs of income taxation? a. Lifeblood doctrine of taxation b. Symbiotic relation in taxation Downloaded by John Paul Magbitang (johnpaulmagbitang04@gmail.com) lOMoARcPSD|15752181
  • 3. c. Compensatory purpose of taxation d. Sumptuary purpose of taxation DONOR’S TAX 7. Which of the following transactions is deemed a taxable gift? a. Condonation or remission of a debt b. Sale of residential house and lot for less than adequate and full consideration in money or money’s worth c. Both (a) and (b) d. Neither (a) nor (b) 8. Which of the following statements relative to donor’s tax is false? a. The spouses shall file separate donor’s tax returns where the thing donated is common property. b. Each parent shall be entitled to the P10,000 exemption on account of marriage of a child. c. Exemptions and deductions cannot be claimed where the 30% tax rate on stranger is applicable. d. None of the foregoing. 9. The spouses Esme and Carlisle wanted to donate a parcel of land to their son Edward who is getting married in December, 2016. The parcel of land has a zonal valuation of P420,000.00. What is the most efficient mode of donating the property? a. The spouses should first donate in 2016 a portion of the property valued at P20,000, then spread the P400,000 equally for 2017, 2018, 2019 and 2020. b. Spread the donation over a period of 5 years by the spouses donating P100,000 each year from 2016 to 2020. c. The spouses should each donate a P110,000 portion of the value of the property in 2016 then each should donate P100,000 in 2017. d. The spouses should each donate a P100,000 portion of the value of the property in 2016, and another P100,000 each in 2017. Then, in 2018, Esme should donate the remaining P20,000. 10. Exempted from donor’s taxation are gifts made a. For the use of the barangay. b. In consideration of marriage. c. To a school which is a stock corporation. d. To a for-profit government corporation. 11. Caroline donated P110,000.00 to her friend Vicky who was getting married. Caroline gave no other gift during the calendar year. What is the donor's tax implication on Caroline’s donation? a. The P100,000 portion of the donation is exempt since given in consideration of marriage. b. A P10,000 portion of the donation is exempt being a donation in consideration of marriage. Downloaded by John Paul Magbitang (johnpaulmagbitang04@gmail.com) lOMoARcPSD|15752181
  • 4. c. Caroline shall pay a 30% donor's tax on the P110,000 donation. d. The P100,000.00 portion of the donation is exempt under the rate schedule for donor's tax. 12. A non-stock, non-profit school always had cash flow problems, resulting in failure to recruit well-trained administrative personnel to effectively manage the school. In 2017, Don Leon donated P100 million pesos to the school, provided the money shall be used solely for paying the salaries, wages, and benefits of administrative personnel. The donation represents less than 10% of Don Leon's taxable income for the year. Is he subject to donor's taxes? a. No, since the donation is actually, directly, and exclusively used for educational purposes. b. Yes, because the donation is to be wholly used for administration purposes. c. Yes, since he did not obtain the requisite NGO certification before he made the donation. d. No, because the donation does not exceed 10% of his taxable income for 2017. 13. What law shall govern the imposition of donor’s tax? a. The law in force at the time of perfection of the donation b. The law in force at the time of completion of the donation c. The law in force at the time of perfection or completion depending upon the agreement of the parties d. None of the choices 14. Andy, married, donated a land commonly owned by him and her spouse worth P500,000 to her friend Joan. Only Andy signed the deed of donation. Joan assumed P200,000 unpaid mortgage on the property. How much is the donor’s tax due? a. P6,000 c. P1,000 b. P90,000 d. P45,000 500k - 200k = 300k x 30% = 90,000 15. Dondie, resident citizen, made the following donations on April 28 of the current year:  To his sister, Donna,P175,000 worth of property situated in Paris, France. The donor’s tax paid is P40,000.  To Dara, his girlfriend in the Philippines, jewelry valued at P225,000.  To International Rice Research Institute, cash amounting to P50,000. The donor’s tax due after tax credit is – a. P69,000 c. P29,000 b. P38,813 d. P5,625 Downloaded by John Paul Magbitang (johnpaulmagbitang04@gmail.com) lOMoARcPSD|15752181
  • 5. 225,000 x 30% = 67,500 + (175,000 - 100,000 x 2% Tabular) = 69,000 - (69,000 x 175k/400k) = 38,813 ESTATE TAX 16. Which of the following is not a part of the gross estate? a. Conjugal property b. Community property c. Exclusive property of the decedent d. Exclusive property of the surviving spouse 17. Who among the following transferors is not liable for estate tax on the property transferred during his lifetime? a. The testator who bequeaths property to his heirs in a last will and testament executed and probated during his lifetime b. The donor who reserves his right to amend or revoke the donation of property in favor of the donee c. The donee of an appointed property who is required under a power of appointment to transfer such property upon death to his eldest child d. The transferor of personal property who sold it for insufficient consideration 18. Which of the following properties of the spouses will be part of common properties under a regime of Conjugal Partnership of Gains? a. Land inherited during the marriage b. Fruits of land inherited c. Jewelry inherited during the marriage d. Building donated before marriage 19. Which of the following items is not considered as a “special deduction” in computing the taxable net estate of the decedent? a. Vanishing deduction b. Medical expenses c. Standard deduction d. Family home allowance 20. When the payment of estate tax will cause undue hardship upon the heirs or the estate which is undergoing judicial settlement before the court, the BIR Commissioner may grant an extension for a period not exceeding: a. 5 years b. 3 years c. 2 years d. 1 year Next three (3) questions are based on the following: Orland, married resident citizen, died on August 20, 2016. The estate reported the following assets and deductions: Conjugal Properties: Fishpond, Bulacan P1,500,000 Family Home, Makati 1,500,000 Cash in bank 900,000 Downloaded by John Paul Magbitang (johnpaulmagbitang04@gmail.com) lOMoARcPSD|15752181
  • 6. Exclusive Properties of Orland: Land, inherited from his father who died on July 20, 2012. The value of land at the time of inheritance was P210,000. The land was mortgaged for P30,000 which was unpaid at the time of death of his father,P10,000 of which was paid by Orland before he died. P400,000 Land, donated on February 14, 2013 by his mother who died on November 2, 2014. The value of the land when donated was P500,000 while upon death of his mother was P400,000. 600,000 Exclusive Properties of Wife: Farm in Laguna, acquired before marriage 2,000,000 Deductions claimed: Funeral expenses 250,000 Fire loss of apartment (occurred 4 months after death) 80,000 Bad debts (represents unpaid receivable from Bert, an insolvent) 100,000 Mortgage on inherited land 30,000 Vanishing deduction on inherited land 40,000 Vanishing deduction on donated land 20,000 Standard deduction 2,000,000 21. The gross estate is: a. P4,000,000 c. P7,000,000 b. P4,710,000 d. P5,000,000 22. The vanishing deduction is: a. P184,000 c. P255,760 b. P220,800 d. P292,560 23. The net taxable estate is: a. P819,200 c. P829,200 b. P804,200 d. P579,200 Downloaded by John Paul Magbitang (johnpaulmagbitang04@gmail.com) lOMoARcPSD|15752181
  • 7. SOLUTION: Properties Exclusive Conjugal Total Fishpond, Bulacan P1,500,000 1,500,000 Family Home, Makati 1,500,000 1,500,000 Cash in bank 900,000 900,000 Land, inherited from Father died on July 20, 2012. 400,000 400,000 Land, Donated from Mother on Feb 14, 2013. 600,000 600,000 Claims to Insolvent 100,000 100,000 GROSS ESTATE 1,000,000 4,000,000 5,000,000 Allowable Deductions: Funeral Expenses (200,000) (200,000) Fire Loss (80,000) (80,000) Bad Debts (100,000) (100,000) Mortgage on Land (20,000) (20,000) Vanishing Deductions* (220,800) (220,800) Net Estate before Special Deductions 759,200 3,620,000 4,379,200 Special Deductions: Family Home (1,500,000 x 1/2) (750,000) Special Deduction (1,000,000) Share of Surviving Spouse (1,810,000) NET TAXABLE ESTATE 819,200 *VANISHING DEDUCTIONS: Land Inherited by Father: Value at the time of Death 210,000 Less: Mortgage Paid (10,000) Initial Basis 200,000 Pro rata: 200/5000 x 400,000 (16,000) Final Basis 184,000 Vanishing Rate (4 years but not more 5 yrs) 20% Vanishing Deduction 36,800 Land Donated by Mother: Value at the time of Donation 500,000 Initial Basis 500,000 Pro rata: 500/5000 x 400,000 (40,000) Final Basis 460,000 Vanishing Rate (3 years but not more 4 yrs) x 40% Vanishing Deduction 184,000 TOTAL VANISHING DEDUCTIONS 220,800 **If TRAIN Law is applied the Estate tax is P 49,152 (P819,200 x 6%) Downloaded by John Paul Magbitang (johnpaulmagbitang04@gmail.com) lOMoARcPSD|15752181
  • 8. 24. Abe, married resident alien, died on January 15, 2017. She left the following properties, expenses and obligations: Community properties, Philippines (including family home valued at P1,800,000) P5,000,000 Community properties, Abroad 2,000,000 Exclusive properties, Philippines 3,000,000 Actual funeral expenses 300,000 Judicial expenses 200,000 Medical expenses (incurred w/in 1yr. before death) 600,000 Devise to National Gov’t 50,000 Legacy to Local Gov’t 70,000 The net taxable estate is: a. P3,780,000 c. P3,580,000 b. P3,680,000 d. P3,530,000 RESIDENT ALIEN Particulars Exclusive Community Total All Properties w/i & w/o 3,000,000 7,000,000 10,000,000 Funeral Expense (200,000) (200,000) Judicial Expense (200,000) (200,000) Transfers (50,000 + 70,000) (120,000) (120,000) Gross Estate 2,880,000 6,600,000 9,480,000 Share of Surviving Spouse (3,300,000) Medical Expenses (500,000) Family Home (1/2 of 1,800,000) (900,000) Standard Deductions (1,000,000) Net Estate 3,780,000 25. Based on the above problem, if the decedent is a non-resident alien, how much is the net taxable estate? a. P2,755,000 c. P5,220,000 b. P2,880,000 d. P5,380,000 NON - RESIDENT ALIEN Particulars Exclusive Community Total All Properties w/i Only 3,000,000 5,000,000 8,000,000 Funeral Expense 200,000 Judicial Expense 200,000 Allowable Deduction 400,000 x 8M/10M (320,000) (320,000) Transfers(50,000 + 70,000) (120,000) (120,000) Gross Estate 2,880,000 4,680,000 7,560,000 Share of Surviving Spouse (2,340,000) Net Estate 5,220,000 26. Arthur, Filipino, married died leaving the following estate: Car acquired before marriage by Arthur P 300,000 Downloaded by John Paul Magbitang (johnpaulmagbitang04@gmail.com) lOMoARcPSD|15752181
  • 9. Car acquired before marriage by wife 450,000 House and lot acquired during marriage 1,500,000 Jewelries of wife 100,000 Personal properties inherited by Arthur during marriage 250,000 Benefits from SSS 50,000 Retirement benefits 150,000 Proceeds of group insurance taken by his employer 75,000 Land inherited by the wife during marriage 1,000,000 Income earned from the land inherited by wife (25% of which was earned after death) 200,000 16. How much is the gross estate if the property relationship is conjugal partnership of gains is: a. P2,600,000 c. P1,950,000 b. P3,600,000 d. P2,200,000 Car acquired before marriage by Arthur P 300,000 House and lot acquired during marriage 1,500,000 Personal properties inherited by Arthur during marriage 250,000 Income earned from the land inherited by wife (25% of which was earned after death) 150,000 Gross Estate 2,200,000 27. Based on the preceding number, the gross estate if the property relationship is absolute community of property is: a. P2,600,000 c. P1,950,000 b. P3,600,000 d. P2,500,000 Car acquired before marriage by Arthur P 300,000 Car acquired before marriage by wife 450,000 House and lot acquired during marriage 1,500,000 Jewelries of wife 100,000 Personal properties inherited by Arthur during marriage 250,000 Gross Estate 2,600,000 VAT 28. LBJ made the following sales during the 12-month period: Sales, VAT taxable transactions P1,500,000 Sales, VAT zero-rated transactions 400,000 Sales, VAT exempt transactions 100,000 Total P2,000,000 Which of the following statements is correct? a. LBJ may not register under the VAT system because his sales from VAT taxable transactions did not exceed P1,919,500. b. LBJ may not register under the VAT system because his sales from VAT taxable and zero-rated transactions did not exceed P1,919,500. Downloaded by John Paul Magbitang (johnpaulmagbitang04@gmail.com) lOMoARcPSD|15752181
  • 10. c. LBJ is required to register because his total 12-month sales exceeded P1,919,500. d. None of the foregoing. 29. Which of the following is exempt from VAT? a. Common carriers transporting passengers by air within the Philippines b. Common carriers transporting passengers by sea within the Philippines c. Common carriers transporting passengers by land within the Philippines d. Common carriers transporting cargoes by air within the Philippines 30. Which statement is correct about value-added tax on goods or properties sold? a. It is based on gross sales and not on net sales; b. May be due even if the goods or properties were not actually sold; c. Does not cover goods exported; d. It forms part of the selling expense of the trader. 31. For value-added tax purposes, which of the following transactions of a VAT-registered taxpayer may not be zero-rated? a. Export sales b. Foreign currency denominated sales c. Sale of goods to the Asian Development Bank d. Sale of goods to an export oriented enterprise 32. A subdivision developer sold five (5) residential house and lots, each to different vendees, for P3,000,000 per lot, or a total sales of P15,000,000 for the taxable period. These sales shall be classified as: a. 12% VAT transactions b. 0% VAT transactions c. VAT exempt transactions d. None of the foregoing **3,199,000 each is the threshold 33. CP operated a retail business that had been generating sales not exceeding the threshold for VAT exempt persons. However, he desires to be registered under the VAT system for the first time in order to benefit from input tax credits. What benefit may CP be entitled to once he registers under the VAT system? a. Tax refund b. Presumptive input tax credit c. Transitional input tax credit d. None of the foregoing Downloaded by John Paul Magbitang (johnpaulmagbitang04@gmail.com) lOMoARcPSD|15752181
  • 11. 34. What institution is required to deduct and withhold a final VAT of 5% on the purchase of goods or services subject to VAT? a. National government or any political subdivision thereof b. Government-owned or controlled corporations c. Both (a) and (b) d. Neither (a) nor (b) 35. In the value-added tax on sale of services, the output tax is computed: a. On the billings of the month b. On collections of the month on all billings made c. On the contract price of contracts completed during the taxable period d. Only and strictly on labor performed under the contract for services 36. Which statement is wrong? a. There is a transitional input tax from purchases of goods or properties; b. There is a transitional input tax from purchases of services; c. There is a transitional input tax from purchases of materials; d. There is a transitional input tax from purchases of supplies. 37. Which of the following statements is correct on the inventory balance in the financial statements at any given date of a VAT- registered person? a. Balance, net of input taxes b. Balance, inclusive of input taxes c. Balance on which the transitional input tax is computed annually d. Balance where the VAT thereon may be calculated by multiplying it by 12% 38. Genson Distribution Inc., a VAT taxpayer, had the following data in a month: Cash sales P200,000 Open account sales 500,000 Consignment: 0 to 30 days old (on which there were remittances from consignees of P200,000) 600,000 31 to 60 days old 700,000 61 days old and above 900,000 How much is the output tax? a. P348,000 c. P264,000 b. P216,000 d. P108,000 Downloaded by John Paul Magbitang (johnpaulmagbitang04@gmail.com) lOMoARcPSD|15752181
  • 12. Cash sales P200,000 Open account sales 500,000 Consignment: 0 to 30 days old (on which there were remittances from consignees of P200,000) 200,000 61 days old and above 900,000 Total VATABLE SALES 1,800,000 VAT RATE 12% OUTPUT VAT 216,000 42. The financial records of Benz Corp., a VAT-registered taxpayer, for the taxable year 2016 disclosed the following: Local sales to private entities 1,500,000 Export Sales 500,000 Local sales to government 800,000 How much is the total sales subject to value-added tax? a. P2,800,000 c. P2,000,000 b. P2,300,000 d. P1,500,000 Local sales to private entities 1,500,000 Export Sales 500,000 Local sales to government 800,000 Total VATABLE SALES 2,800,000 43. Mantika Corp., a VAT-registered Corp., is a producer of cooking oil from coconut and corn. It had the following data for the month of January 2017: Sales, gross of VAT P 784,000 Corn & Coconut, 12-31-16 50,000 Purchases of Corn & Coconut 330,000 Corn & Coconut, 1-31-17 20,000 Purchases from VAT suppliers, VAT included: Packaging Materials 56,000 Supplies 16,800 The value-added tax payable for the month: a. P56,060 c. P60,650 b. P54,900 d. P63,000 Sales, gross of VAT P 784,000 Output TAX 84,000 Purchases of Corn & Coconut (330,000 x4%) (13,200) Purchases from VAT suppliers, VAT included: Packaging Materials 56,000 Supplies 16,800 72,800 x3/28 (7,800) VAT PAYABLE 63,000 Downloaded by John Paul Magbitang (johnpaulmagbitang04@gmail.com) lOMoARcPSD|15752181
  • 13. 44. Bunga Inc., a VAT taxpayer, is engage in the business of processing of fruits. Its data on sales and purchases for the month of August are provided below: Sales P200,000 Purchases: Fresh Fruits 30,000 Raw sugarcane 12,000 Tin Can, gross of VAT 12,320 Paper Labels, net of VAT 5,000 Cardboard for boxes, net of VAT 8,000 Freight, gross of VAT (50% still unpaid) 10,080 How much is the value-added tax payable? a. P20,580 c. P19,380 b. P18,900 d. P20,100 Sales P200,000 Output Tax 24,000 Purchases: Fresh Fruits 30,000 Raw sugarcane 12,000 Tin Can, gross of VAT 12,320 Input Tax (1,320) Paper Labels, net of VAT 5,000 Input Tax ( 600) Cardboard for boxes, net of VAT 8,000 Input Tax ( 960) Freight, gross of VAT (50% still unpaid) 10,080 Input Tax ( 540) VAT PAYABLE 20,580 45. Bahay Kubo Inc. is a real estate dealer. Details of its sales during the year showed the following: Date of sale June 2, 2017 Consideration in the deed of sale P 5,000,000 Fair market value in the assessment rolls 4,800,000 Zonal Value 5,200,000 Schedule of payments: June 2, 2017 1,000,000 June 2, 2018 2,000,000 June 2, 2019 2,000,000 How much is the output tax to be recognized for the June 2, 2018 payment? a. P0 c. P249,600 b. P124,800 d. P624,000 **Zonal Value 5,200,000 x 12% = 624,000 x 2M/5M = 249,600 Output Tax for 2018 46. Assuming that the scheduled payment on June 2, 2017 is P2,000,000, how much is the output tax to be recognized for the June 2, 2019 payment? a. P0 c. P249,600 b. P124,800 d. P624,000 Downloaded by John Paul Magbitang (johnpaulmagbitang04@gmail.com) lOMoARcPSD|15752181
  • 14. Zero as in 0 for the sale will no longer qualify as Installment Sales. 47. Mr. Karpentero, a vat-registered building contractor, has the following data on gross receipts in a month, any tax not included:  From Mr. A, a private property owner, final payment on the contract price, net of 5% agreed retention fee of P2,850,000  From Mr. B, a payment of 5% retention on the contract price previously made by him P100,000  From Mr. C, for materials used in the construction 500,000 How much is the output tax? a. P414,000 c. P72,000 b. P342,000 d. P62,000 Final Payment on Contracts 2,850,000 Retention 100,000 Materials 500,000 TOTAL Receipts 3,450,000 VAT Rate 12% Output VAT 414,000 48. COC Inc., in its first month of operation, and as a VAT taxpayer, purchased various fixed assets. Purchases of fixed assets in the first month were as follows: Light equipment, with a useful life of 3 years P 300,000 Heavy equipment, with a useful life of 10 years 4,000,000 How much is the input tax available for the month? a. P516,000 c. P480,000 b. P9,000 d. P8,600 Light Equipment 300,000/36 x 12% 1,000 Heavy Equipment 4,000,000/60 x 12% 8,000 Total Input VAT 9,000 49. Kusina Co., had its kitchen assembled by a VAT taxpayer. It took six months for the contractor to finish the work. Kusina Co. purchased materials in July from VAT suppliers at a cost of P500,000, VAT not included. Payment to the contractor in July 2017 on the Construction in Progress, VAT not included was: On contractor’s billing in June P100,000 On contractor’s billing in July 70,000 Downloaded by John Paul Magbitang (johnpaulmagbitang04@gmail.com) lOMoARcPSD|15752181
  • 15. The input tax available in July is: a. P0 c. P60,000 b. P80,400 d. P20,400 Materials from VAT Supplier 500,000 On contractor’s billing in June 100,000 On contractor’s billing in July 70,000 Total 670,000 VAT Rate 12% Output Tax 80,400 50. Data from the books of accounts of a VAT taxpayer for February: Domestic Exports Sales P 2,000,000 8,000,000 Purchases: From VAT Suppliers: Goods for sale 600,000 2,400,000 Supplies & services 90,000 360,000 From Suppliers Paying percentage tax: Goods for sale 100,000 1,500,000 Supplies & services 20,000 80,000 If the input taxes attributable to zero-rated sales are claimed as tax credit, the net value-added tax refundable is: a. P136,000 c. P145,000 b. P203,924.70 d. P174,000 Output Tax (2,000,000 x 12%) 240,000 Input Tax on Domestic Sales (690,000 x 12%) (82,800) Input Tax on Zero Rated (2,760,000 x 12%) (331,200) Refundable VAT (174,000) INCOME TAXATION 51. C. Lee, Chinese national, arrived in the Philippines on January 1, 2012 to visit his Filipina paramour. He planned to stay in the country until December 31, 2016, by which time he would go back to his legal wife and family in China. C. Lee derived income during his stay here in the Philippines. For the taxable year 2012, C. Lee shall be classified as a: a. Resident alien b. Non-resident alien engaged in trade or business in the Philippines c. Non-resident alien not engaged in trade or business in the Philippines d. Special alien employee 52. The following individual taxpayers are subject to the graduated income tax rates of 5%-32%, except a. Filipino citizens b. Resident aliens c. Non-resident alien engaged in trade or business in the Philippines Downloaded by John Paul Magbitang (johnpaulmagbitang04@gmail.com) lOMoARcPSD|15752181
  • 16. d. Non-resident alien not engaged in trade or business in the Philippines 53. In which of the following cases will the dividend income from a foreign corporation be classified as “income without” a. Less than 50% of the foreign company’s gross income for the preceding three (3) years prior to the dividend declaration was derived from sources within the Philippines. b. 50% of the foreign company’s gross income for the preceding three (3) years prior to the dividend declaration was derived from sources within the Philippines. c. More than 50% of the foreign company’s gross income for the preceding three (3) years prior to the dividend declaration was derived from sources within the Philippines. d. Always classified as income without”. 54. D’ Lion, Inc., a Philippine corporation, sold through the local stock exchange 10,000 PLDT shares that it bought 2 years ago. D’ Lion sold the shares for P2 million and realized a net gain of P200,000. How shall it pay tax on the transaction? a. It shall declare a P2 million gross income in its income tax return, deducting its cost of acquisition as an expense. b. It shall report the P200,000 in its corporate income tax return adjusted by the holding period. c. It shall pay 5% tax on the first P100,000 of the P200,000 and 10% tax on the remaining P100,000. d. It shall pay a tax of one-half of 1% of the P2 million gross sales. 55. The payor of passive income subject to final tax is required to withhold the tax from the payment due the recipient. The withholding of the tax has the effect of a. A final settlement of the tax liability on the income. b. A credit from the recipient's income tax liability. c. Consummating the transaction resulting in an income. d. A deduction in the recipient's income tax return. 56. Winterfell, Inc., bought a parcel of land in 2015 for P7 million as part of its inventory of real properties. In 2017, it sold the land for P12 million which was its zonal valuation. In the same year, it incurred a loss of P6 million for selling another parcel of land in its inventory. These were the only transactions it had in its real estate business. Which of the following is the applicable tax treatment? a. Winterfell shall be subject to a tax of 6% of P12 million. b. Winterfell could deduct its P6 million loss from its P5 million gain. c. Winterfell’s gain of P5 million shall be subject to the holding period. d. Winterfell's P6 million loss could not be deducted from its P5 million gain. Downloaded by John Paul Magbitang (johnpaulmagbitang04@gmail.com) lOMoARcPSD|15752181
  • 17. 57. Passive income includes income derived from an activity in which the earner does not have any substantial participation. This type of income is a. Usually subject to a final tax. b. Exempt from income taxation. c. Taxable only if earned by a citizen. d. Included in the income tax return. 58. In 2017, Alice earned P500,000 as income from her beauty parlor and received P250,000 as Christmas gift from her aunt. She had no other receipts for the year. She spent P150,000 for the operation of her beauty parlor. For tax purposes, her gross income for 2017 is a. P750,000 c. P350,000 b. P500,000 d. P600,000 59. Which of the following items is not part of gross income to be reported in the income tax return? a. Increase in value of land b. Gambling winnings c. Prize of P10,000 d. Gain from sale of store’s air conditioner 60. Mr. Yu leased his lot to Mr. Uy. The contract calls for Mr. Uy to construct a house which would serve as the residence of the latter, the ownership thereof to be vested in Mr. Yu after the expiration of the lease. When the house was completely constructed, the remaining term of the lease was 10 years. The residential house had an estimated useful life of 15 years. What is the tax implication of the leasehold improvement? a. Mr. Yu derives taxable income on the improvement; Mr. Uy can claim depreciation expense as a deduction from gross income. b. Mr. Yu derives taxable income on the improvement; Mr. Uy cannot claim depreciation expense as a deduction from gross income. c. Mr. Yu does not derive taxable income on the improvement; Mr. Uy cannot claim depreciation expense as a deduction from gross income. d. Mr. Yu does not derive taxable income on the improvement; Mr. Uy can claim depreciation expense as a deduction from gross income. 61. Assume the same facts in the immediately preceding number, except that at the time of the completion of the residential house, the remaining term of the lease was 15 years while the useful life of the house was 10 years. What is the tax implication of the leasehold improvement? a. Mr. Yu derives taxable income on the improvement; Mr. Uy can claim depreciation expense as a deduction from gross income. b. Mr. Yu derives taxable income on the improvement; Mr. Uy cannot claim depreciation expense as a deduction from gross income. Downloaded by John Paul Magbitang (johnpaulmagbitang04@gmail.com) lOMoARcPSD|15752181
  • 18. c. Mr. Yu does not derive taxable income on the improvement; Mr. Uy cannot claim depreciation expense as a deduction from gross income. d. Mr. Yu does not derive taxable income on the improvement; Mr. Uy can claim depreciation expense as a deduction from gross income. 62. Which of the following expenses may be deducted from gross compensation income? a. Depreciation of permanent assets b. Premium payments on health and/or hospitalization insurance c. Bad debts written off d. Optional standard deduction 63. Which of the following items of interest expense may be deducted from gross income? a. Interest on corporation’s preferred stock b. Interest on loan for construction of a rest house c. Interest for delinquency in the payment of percentage tax d. Interest on bank loan to finance petroleum exploration 64. Which of the following taxes may be deducted from gross income? a. Percentage tax on sale of listed stock b. Business permit fee paid to the city government c. Income tax d. Tax on interest on bank deposit 65. Who among the following taxpayers may not claim a tax credit or deduction on income tax paid to foreign countries? a. Resident citizens b. Resident aliens c. Domestic corporations d. General Co-Partnerships 66. The loss from sale or exchange of property is deductible from gross income where the sale or exchange is: a. Between fiduciary of a trust and the fiduciary of another trust if they have the same grantor b. Between fiduciary of a trust and the beneficiary of such trust c. Between an individual and his first cousin d. Between an individual and a corporation if the former owns more than 50% in value of the outstanding capital stock of the latter 67. Anne, claimed a bad debt of P50,000 as a deductible expense in the taxable year 2016. In 2017, Anne was able to recover the P50,000 debt already written off in the preceding year. What is the treatment for tax purposes of the recovery of the bad debt? a. Report the recovery of the bad debt as gross income in 2016. b. Report the recovery of the bad debt as gross income in 2017. Downloaded by John Paul Magbitang (johnpaulmagbitang04@gmail.com) lOMoARcPSD|15752181
  • 19. c. Disregard the recovery of the bad debt. d. Amend the 2016 income tax return to rectify the deduction for bad debt claimed. 68. Which of the following assets shall be subject to depletion? a. Machinery b. Land containing ore deposit c. Commercial d. Goodwill 69. Which of the following individual taxpayers may claim basic and additional personal exemptions for income tax purposes? a. Non-resident aliens engaged in trade or business in the Philippines, in the absence of reciprocity b. Non-resident aliens not engaged in trade or business in the Philippines c. Both (a) and (b) d. Neither (a) nor (b) 70. Harry works as financial consultant in an oil firm in Dubai. Aside from his salary thereat, he also maintains a 10-door apartment in Manila which he inherited from his parents when he was already married. On the other hand, Wilma, his wife, is employed as a loan officer at a local bank. Data pertaining to their dependents appear below for the taxable year 2017:  Anton - Son who turned 23 on April 1, 2017; incapable of self-support due to loss of both legs in an accident;  Bunny - 21 year old daughter who is taking up culinary arts in Paris, France;  Charlie - 15-year old adulterous son of Harry living with the couple;  Dina - 12-year old child who died from a vehicular accident on January 1, 2017; and  Evan - 80-year old father of Wilma, supported by her and living with the couple. The basic and additional personal exemptions of Harry for the taxable year 2017 amounts to: a. P50,000 and P100,000, respectively b. P50,000 and P75,000, respectively c. P50,000 and P0, respectively d. P0 and P0, respectively 71. Assume the same facts above, the basic and additional personal exemptions of Wilma for the taxable year 2016 amount to: a. P50,000 and P100,000, respectively b. P50,000 and P75,000, respectively c. P50,000 and P0, respectively d. P0 and P0, respectively Downloaded by John Paul Magbitang (johnpaulmagbitang04@gmail.com) lOMoARcPSD|15752181
  • 20. 72. Which of the following statements does not characterize a capital asset? a. It may be real or personal property. b. It is not always subject to a holding period. c. It is normally subject to value-added tax when it is sold. d. It is not always subject to a final tax. 73. Which of the following transactions is exempt from capital gains tax? a. The sale of the principal residence of the taxpayer where the entire proceeds is used to purchase a vacation lot at Tagaytay b. The sale of a beach lot of the taxpayer where the entire proceeds is used to construct his principal residence c. The sale of the principal residence of the taxpayer for the second time in ten (10) years to purchase another principal residence d. None of the choices 74. Which of the following transactions is treated as a capital asset transaction for income tax purposes? a. Sale of a residential lot by a subdivision developer b. Sale of a used delivery truck by a retailing company c. Liquidation of partnership business d. Sale of shares of stock by a dealer in securities 75. Which of the following is not an attribute of a deferred-payment sale? a. The initial payments exceed 25% of the selling price in the year of sale. b. The obligations or promissory notes received by the vendor from the vendee are considered as equivalent to cash. c. The tax may be paid in installments. d. The sale involves both real and personal property. 76. The deductible expenses of an estate may consist of: a. Deductible expenses allowed to an individual taxpayer b. Income distributed to beneficiaries c. Both (a) and (b) d. Neither (a) nor (b) 77. Determine which of the following trusts shall the taxable income be consolidated and the income tax thereon computed on the basis of such consolidated income? a. Trust No. 1 and Trust No. 2 have the same grantor and with different beneficiaries. b. Trust No. 1 and Trust No. 2 have the same grantor and the same beneficiary. c. Trust No. 1 and Trust No. 2 have different grantors and the same beneficiary. d. Trust No. 1 and Trust No. 2 have different grantors and the same fiduciary and beneficiary. Downloaded by John Paul Magbitang (johnpaulmagbitang04@gmail.com) lOMoARcPSD|15752181
  • 21. 78. Inday is a resident citizen of the Philippines. Data for a year: Gross income from business P 700,000 Royalty from books 40,000 Gain on direct sale to buyer of shares of stock of a domestic corporation held as capital asset 70,000 Loss on sale of land in the Philippines held as capital asset with cost of P1,500,000 when the zonal value is P1,200,000 500,000 Business Expenses 300,000 How much is the total income tax expense for the year? a. P177,500 c. P159,500 b. P80,000 d. P156,000 Gross Income from Business 700,000 Less: Business Expense 300,000 Personal Exemptions 50,000 350,000 Net Taxable income 350,000 Tabular Schedule: 1st 250,000 Tax is 50,000 Excess of 250,000 - 350,000 = 100,000 x 30% 30,000 Capital Gains Tax on Shares (70,000 x 5%) 3,500 Royalty Income (40,000 x 10%) 4,000 Loss on Sale (Zonal Value 1.2 x 6%) 72,000 Total Income Tax Expense 159,500 79. Mercy is a citizen and resident of the Philippines. She had a compensation income (net of exclusions) of P200,000 and a net income from business of P700,000 for a year. She made quarterly income tax payments amounting to P237,000 and her employer withheld P25,000 on her compensation income. The income tax payable (refundable) for the year is: a. (P25,000) c. (P42,000) b. P237,000 d. (P37,500) Compensation Income 200,000 Net Income from Business 700,000 Total Income 900,000 Basic Exemptions (50,000) Taxable Income 850,000 1st 500,000 Tax 125,000 Excess of 500,000- 850,000 x 32% 112,000 Total Tax Due 237,000 Less: Taxes Payments Quarterly Payments 237,000 Compensation CWT 25,000 262,000 Refundable /Creditable Tax (25,000) Downloaded by John Paul Magbitang (johnpaulmagbitang04@gmail.com) lOMoARcPSD|15752181
  • 22. 80. EMT has the following data on his passive income earned during the year 2016: Philippines Abroad Interest income from bank deposits 45,000 25,000 Interest income from FCDU 50,000 -0- Royalties from books 20,000 30,000 Royalties from computer programs 20,000 40,000 Dividend income from a domestic corporation 27,000 13,000 Dividend income from a foreign corporation 33,000 22,000 How much is the final withholding tax if the taxpayer is a resident citizen? a. P21,450 c. P17,700 b. P20,400 d. P36,250 Philippines Interest income from bank deposits 45,000 x 20% = 9,000 Interest income from FCDU 50,000 x 7.5% = 3,750 Royalties from books 20,000 x 10% = 2,000 Royalties from computer programs 20,000 x 20% = 4,000 Dividend income from a domestic corporation 27,000 x 10% = 2,700 Total Withholding Taxes 21,450 81. How much is the final withholding tax if the taxpayer is a resident alien? a. P21,450 c. P17,700 b. P20,400 d. P36,250 SAME COMPUTATION AS RESIDENT CITIZEN 82. How much is the final withholding tax if the taxpayer is a non- resident citizen? a. P21,450 c. P17,700 b. P20,400 d. P36,250 Philippines Interest income from bank deposits 45,000 x 20% = 9,000 Royalties from books 20,000 x 10% = 2,000 Royalties from computer programs 20,000 x 20% = 4,000 Dividend income from a domestic corporation 27,000 x 10% = 2,700 Total Withholding Taxes 17,700 83. How much is the final withholding tax if the taxpayer is a non- resident alien engaged in trade or business? a. P21,450 c. P17,700 b. P20,400 d. P36,250 Downloaded by John Paul Magbitang (johnpaulmagbitang04@gmail.com) lOMoARcPSD|15752181
  • 23. Philippines Interest income from bank deposits 45,000 x 20% = 9,000 Royalties from books 20,000 x 10% = 2,000 Royalties from computer programs 20,000 x 20% = 4,000 Dividend income from a domestic corporation 27,000 x 20% = 5,400 Total Withholding Taxes 20,400 84. How much is the final withholding tax if the taxpayer is a non- resident alien not engaged in trade or business? a. P21,450 c. P17,700 b. P20,400 d. P36,250 Interest income from bank deposits 45,000 Royalties from books 20,000 Royalties from computer programs 20,000 Dividend income from a domestic corporation 27,000 Dividend income from a foreign corporation 33,000 TOTAL PASSIVE INCOME 145,000 Final Withholding Tax Rate 25% Total Withholding Taxes 36,250 85. Nonoy is an employee of a firm in Quezon City. He is supporting his 4 year old brother who is living with him. Data on his compensation income for the year shows: Regular Salary P 240,000 Thirteenth month pay 20,000 Quarterly bonus 40,000 Payroll Deductions: SSS Premiums 3,000 Philhealth contributions 1,200 Pagibig contributions 4,000 Labor union dues 1,000 Premium payments on hospitalization insurance 3,000 Payment of loan 5,000 86. How much is the taxable income? a. P210,800 c. P208,400 b. P207,800 d. P260,800 Regular Salary P 240,000 Total Income 240,000 Less: Basic Exemption (50,000) SSS Premiums (3,000) Philhealth contributions (1,200) Pagibig contributions (4,000) Labor union dues (1,000) Net Taxable Income 180,800 +30,000 if old laws = 218,000 Downloaded by John Paul Magbitang (johnpaulmagbitang04@gmail.com) lOMoARcPSD|15752181
  • 24. 87. A domestic corporation, in its fifth (5th ) year of operations, had the following data for the year: Net sales P 2,000,000 Capital gain on direct sale to a buyer of shares of a domestic corporation for P500,000 200,000 Capital gain on sale thru a real estate broker of land and building outside the Philippines for P5,000,000 1,000,000 Dividend from a domestic corporation 50,000 Interest on bank deposit 40,000 Cost of sales 600,000 Quarterly corporate income tax paid 190,000 Operating expenses 500,000 The income tax payable upon filing of the annual income tax return is: a. P425,000 c. P570,800 b. P380,000 d. P520,800 Net Sales 2,000,000 Cost of Sales (600,000) Operating Expenses (500,000) Net Income from Ordinary Business 900,000 Add Other Income Broker Outside Phils 1,000,000 Total Taxable Income 1,900,000 Corporate Tax Rate 30% Income Tax Due 570,000 Less Quarterly Income Tax Paid (190,000) Income Tax Due and Payable 380,000 88. The Kultura Foundation of the Philippines, a non-stock, non-profit corporation, organized and operated exclusively to preserve and show-case Philippine cultural practices, music, dances, and folk arts, deriving funding from mostly donations, had the following data for the year: Donations received P 20,000,000 Interest income from bank deposit 100,000 Rent income from properties received as donation (net of 5% withholding tax) 475,000 Expenses related to its rent income 30,000 89. How much is the income tax expense of the Corporation for the year? a. P161,000 c. P172,000 b. P136,000 d. P125,600 Rent Income (Gross Up) 500,000 Less: Expenses (30,000) Net Rent Income 470,000 Downloaded by John Paul Magbitang (johnpaulmagbitang04@gmail.com) lOMoARcPSD|15752181
  • 25. Tax Rate 30% Tax Expense 141,000 Add Interest Income Tax 20,000 Total Income Tax Expense 161,000 90. A tax imposed in the nature of a penalty to the corporation to deter tax avoidance of shareholders who avoid paying the dividends tax on the earnings distributed to them by the corporation. a. Minimum corporate income tax b. Optional corporate income tax c. Improperly accumulated earnings tax d. Capital gains tax 91. First statement: An accumulation of earnings or profits (including undistributed earnings or profits of prior years) is unreasonable if it is not necessary for the purpose of the business, considering all the circumstances of the case. Second statement: The term "reasonable needs of the business" are hereby construed to mean the immediate needs of the business, including reasonably anticipated needs. a. Only the first statement is correct b. Only the second statement is correct c. Both statements are correct d. Both statements are incorrect 92. The Improperly Accumulated Earnings Tax (IAET) is imposed on improperly accumulated taxable income earned starting January 1, 1998 by domestic corporations as defined under the Tax Code and which are classified as closely-held corporations at the rate of: a. twenty percent (20%). c. ten percent (10%). b. fifteen percent (15%). d. five percent (5%). 93. The Improperly Accumulated Earnings Tax shall not apply to which of the following corporations? a. Banks and other non-bank financial intermediaries b. Insurance companies c. Publicly-held corporation d. All of the choices 94. First statement: Once the profits have been subjected to improperly accumulated earnings tax, the same shall no longer be subject to the same tax in later years even if not declared as dividends. Second statement: Profits which have been subjected to improperly accumulated earnings tax when finally declared as dividends shall be subject to tax on dividends. a. Both statements are true b. Both statements are false c. Only the first statement is true d. Only the second statement is true Downloaded by John Paul Magbitang (johnpaulmagbitang04@gmail.com) lOMoARcPSD|15752181
  • 26. 95. To avoid payment of IAET, when must the dividends be declared and paid or issued? a. Not later than one year following the close of the taxable year b. Not later than the 15th day following the close of the taxable year c. Not later than 60th day following the close of the taxable quarter d. None of the choices TAX REMEDIES 96. Rosalie, a compensation income earner, filed her income tax return for the taxable year 2013 on March 30, 2014. On May 20, 2017, Rosalie received an assessment notice and letter of demand covering the taxable year 2013 but the postmark on the envelope shows April 10, 2017. Her return is not a false and fraudulent return. Can Rosalie raise the defense of prescription? a. No. The 3 year prescriptive period started to run on April 15, 2014, hence, it has not yet expired on April 10, 2017. b. Yes. The 3 year prescriptive period started to run on April 15, 2014, hence, it had already expired by May 20, 2017. c. No. The prescriptive period started to run on March 30, 2014, hence, the 3 year period expired on April 10, 2017. d. Yes. Since the 3-year prescriptive period started to run on March 30, 2014, it already expired by May 20, 2017. 97. On March 30, 2012 Emmett Foods, Inc. received a notice of assessment and a letter of demand on its April 15, 2009 final adjustment return from the BIR. Emmett Foods then filed a request for reinvestigation together with the requisite supporting documents on April 25, 2012. On June 2, 2012, the BIR issued a final assessment reducing the amount of the tax demanded. Emmett Foods was satisfied with the reduction, it did not do anything anymore. On April 15, 2017 the BIR garnished the corporation's bank deposits to answer for the tax liability. Was the BIR action proper? a. Yes. The BIR has 5 years from the filing of the protest within which to collect. b. Yes. The BIR has 5 years from the issuance of the final assessment within which to collect. c. No. The taxpayer did not apply for a compromise. d. No. Without the taxpayer’s prior authority, the BIR action violated the Bank Deposit Secrecy Law. 98. Renesmee, Inc. received a notice of assessment and a letter from the BIR demanding the payment of P3 million pesos in deficiency income taxes for the taxable year 2015. The financial statements of the company show that it has been suffering financial reverses from the year 2016 up to the present. Its asset position shows that it could pay only P500,000.00 which it offered as a compromise to the BIR. Which among the following may the BIR Downloaded by John Paul Magbitang (johnpaulmagbitang04@gmail.com) lOMoARcPSD|15752181
  • 27. require to enable it to enter into a compromise with Renesmee, Inc.? a. Renesmee must show it has faithfully paid taxes before 2016. b. Renesmee must promise to pay its deficiency when financially able. c. Renesmee must waive its right to the secrecy of its bank deposits. d. Renesmee must immediately deposit the P500,000 with the BIR. 99. As a rule, within what period must a taxpayer elevate to the Court of Tax Appeals a denial of his application for refund of income tax overpayment? a. Within 30 days from receipt of the Commissioner’s denial of his application for refund. b. Within 30 days from receipt of the denial which must not exceed 2 years from payment of income tax. c. Within 2 years from payment of the income taxes sought to be refunded. d. Within 30 days from receipt of the denial or within two years from payment. 100. What is the effect on the tax liability of a taxpayer who does not protest an assessment for deficiency taxes? a. The taxpayer may appeal his liability to the CTA since the assessment is a final decision of the Commissioner on the matter. b. The BIR could already enforce the collection of the taxpayer's liability if it could secure authority from the CTA. c. The taxpayer's liability becomes fixed and subject to collection as the assessment becomes final and collectible. d. The taxpayer's liability remains suspended for 180 days from the expiration of the period to protest. 101. The taxpayer seasonably filed his protest together with all the supporting documents. It is already July 31, 2017, or 180 days from submission of the protest but the BIR Commissioner has not yet decided his protest. Desirous of an early resolution of his protested assessment, the taxpayer should file his appeal to the Court of Tax Appeals not later than a. August 31, 2017. b. August 30, 2017. c. August 15, 2017. d. August 1, 2017. Downloaded by John Paul Magbitang (johnpaulmagbitang04@gmail.com) lOMoARcPSD|15752181
  • 28. DOCUMENTARY STAMP TAX 102. A newly formed corporation issued shares of stocks to its incorporators for P150,000. The par value of the shares issued is P100,000. How much is the documentary stamp tax? a. P500 c. P750 b. P1,000 d. P1,500 Par Value Divide by 200 x 1 peso = 100,000 / 200 = 500 103. Mr. T invested in shares of stock of Kapisananngmga Sisters Inc. amounting to P100,000 with par value of P80,000. After 2 years, he disposed said shares directly to Mr. B for P230,000. The documentary stamp tax on above transaction is: a. P400 c. P300 b. P1,150 d. P862.50 **80,000 / 200 = 400 x 75% = 300 104. Based on number 97, but assuming the shares are without par value, how much is the documentary stamp tax? a. P500 c. P750 b. P1,000 d. P0.00 **Selling Price - Cost = 230,000 - 80,000 = 150,000 / 200 = 750 105. Continuing number 99, if the shares were subsequently sold for P200,000, how much is the documentary stamp tax? a. P750 c. P1,500 b. P187.50 d. P375 **Doc Stamp Upon Original Issuance 750 x 25% = 187.50 106. Mr. Purisima owns a resthouse in Pampanga acquired by him for P10,750,000. He sold the same to Mr. Apo for P5,000,000. The fair market value at the time of sale per assessor’s office is P10,000,000 while zonal value is P15,000,000. The documentary stamp tax on the transaction is: a. P75,000 c. P150,000 b. P225,000 d. P161,250 **15,000,000/1000 = 15,000 x 15 pesos = 225,000 107. Who is liable to the payment documentary stamp tax? a. Mr. Purisima b. Mr. Apo c. It depends on the agreement of the parties d. It depends on who is benefiting on the transaction Downloaded by John Paul Magbitang (johnpaulmagbitang04@gmail.com) lOMoARcPSD|15752181
  • 29. TAXATION QUIZZER PART 2 1. The BIR is assessing deficiency withholding tax on total Repairs and Maintenance claimed by non-stock non-profit organization. The BIR alleges that foundation failed to withhold the correct amount of the 2% expanded withholding tax due on the Repairs and Maintenance expense. As a CPA assisting the foundation in the BIR tax investigation, you can reason that ___. a. The BIR should have considered the 1% expanded withholding tax instead of the 2%in determining the deficiency tax. b. The foundation is exempt from the tax and hence is not liable to the deficiency withholding tax. c. The foundation is exempt from tax and hence is not required to withhold tax on its expenses. d. The BIR should have considered both the 1% and 2% withholding tax in determining the deficiency tax. 2. Which of the following statement is true? a. Taxes are in the nature if contracts between the taxpayers and the government. b. Taxes and debts are similar nature and character. c. As the general rule, no set-off is allowed against the demands for taxes levied for general or local government purposes. d. In taxation, the personal consent of the individual taxpayers is required. 3. Which of the following is not an attribute of tax? a. It is an enforced contribution on a person taxed. b. It is levied by the executive department of the state. c. It is imposed by the state which has jurisdiction over the person, property, or excises. d. It is generally in money 4. Which of the statement below is grammatically correct? a. The imposition of minimum corporate income tax may be suspended if substantial losses are sustained due to a prolonged labor dispute. b. The imposition of minimum corporate income tax maybe suspend if substantial losses are sustained due to prolonged labor dispute. c. The imposition of minimum corporate income tax may be suspended if substantial losses are sustained due to a prolong disputes. d. The imposition of minimum corporate income tax may be suspended. If substantial loss are sustained due to a prolonged labor dispute. 5. Which of the following statements is correct? a. If what is delegated is tax administration, the delegation is invalid. b. If what is delegated is tax administration, the delegation is valid. c. Tariff powers cannot be delegated to the President. d. As a general rule, taxation can be further delegated. Downloaded by John Paul Magbitang (johnpaulmagbitang04@gmail.com) lOMoARcPSD|15752181
  • 30. 6. The BIR issued a tax assessment against the taxpayer who was not given sufficient time to protest the said assessment. The taxpayer noted that their competitors were issued tax assessments but were given enough time to protest. The BIR violated the _____________ of the constitution. a. Equal protection clause c. Equitability principle b. Due process clause d. Uniformity principle Items 7-10 are based on the following information: On March 30, 2012, XXX, Inc., received a notice of assessment and a letter of demand on its April 15, 2009 final adjustment return from the BIR. XXX, Inc., then filed a request for reinvestigation together with the requisite supporting documents on April 25, 2012. On June 2, 2012, the BIR issued a final assessment reducing the amount of the tax demand. Since XXX, Inc., was satisfied with the reduction, it did not do anything anymore. On April 15, 2017 the BIR garnished the corporation’s bank deposits to answer the liability. 7. Was the BIR action proper? a. No, the taxpayer did not apply for the compromise. b. Yes, the BIR has 5years from the filing within which to collect. c. No, without the taxpayer’s prior authority, the BIR action violated the Bank Deposit Secrecy Law. d. Yes, the BIR has 5 years from the issuance if the final assessment within which to collect. 8. What is the effect of the XXX, Inc.’s failure to file a protest on its assessed deficiency taxes? a. The taxpayer may file a motion for reconsideration to the CIR on the matter. b. The taxpayer may appeal his liability to the CTA since the assessment is a final decision of the Commissioner on the matter. c. The taxpayer’s liability becomes fixed and subject to collection as the assessment becomes final and collectible. d. The BIR could already enforced the collection of the taxpayer’s liability if it could secure authority from the CTA. 9. Which statement is correct? The collection of a deficient tax assessment by distraint and levy: a. May be done only once during the taxable year. b. Must be done successively, first by distrait and then by levy; c. May be repeated, if necessary, until the full amount due, including all expenses, is collected; d. Automatically covers the bank deposits of a delinquent taxpayer. 10. Which of the following is grammatically correct? a. Tax assessment refer to the process of determining the correct amount of tax due in accordance with the prevailing tax laws. b. Tax assessment refer to the process of determining the correct amount of tax due in accordance with the prevailed tax laws. Downloaded by John Paul Magbitang (johnpaulmagbitang04@gmail.com) lOMoARcPSD|15752181
  • 31. c. Tax assessment refers to the process of determining the correct amount of tax due in accordance with the prevailed tax laws. d. Tax assessment refers to the process of determining the correct amount of tax due in accordance with the prevailing tax laws. 11. A joker was commissioned in a kiddy party to perform magic. The comic was to be paid P100, 000 for his performance and the parties signed the necessary contract. He then gratuitously assigned his rights under the contract to his son. The son later on collected the P100, 000 talent fee of his father from the contractee. The national internal revenue tax/es payable is/are: a. Income tax only. b. Donor’s tax only. c. Both income and donor’s taxes. d. Neither income tax nor donor’s tax. 12. Which one of the following statements is wrong? a. Income out of the labor of the wife is conjugal property. b. Income out of the separate property of the husband is conjugal property. c. Amount receivable as retirement benefit under R.A. No. 4917 during the marriage is conjugal property. d. Property received is donation when the fair value was P2, 000,000, resulting in a gain of P500, 000. The gain is conjugal property. 13. Amount receivable by the estate of the deceased, his executor or administration as a beneficiary under life insurance policy taken by the decedent upon his own life is: a. Excluded from gross state. b. Part of gross state if the beneficiary is revocable. c. Excluded from gross state if the beneficiary is irrevocable. d. Part of gross state whether the beneficiary is revocable or irrevocable. 14. The following journal entry was made in the purchases journal of a VAT-registered taxpayer: Purchases xxx Cash or Accounts Payable xxx The journal entry signifies that: a. Purchases were from a non-VAT supplier. b. Purchases were from a VAT-registered supplier. c. Purchases were either from non-VAT or VAT-registered supplier. d. Input tax has been taken as part of the cost of purchase. 15. A common carrier by land is engaged in the transport of passengers, goods and cargoes. He is not VAT-registered. What business tax or taxes is he liable to pay? a. 12% value-added tax b. 3% common carrier’s tax Downloaded by John Paul Magbitang (johnpaulmagbitang04@gmail.com) lOMoARcPSD|15752181
  • 32. c. 12% VAT on gross receipts from transport of goods and cargoes and 3% common carrier’s tax on gross receipts from transport of passengers. d. 3% tax on VAT-exempt persons on gross receipts from transport of passengers. 16. Statement 1. As to the property of the state, exemption is the rule and taxation the exception. Statement 2. As of the property of the taxpayer, taxation is the rule and exemption the exception. a. Both statements are correct b. Both statements are incorrect. c. Statement 1 is correct while statement 2 is incorrect. d. Statement 1 is incorrect while statement 2 is correct. 17. Foreign income taxes paid by the resident citizen or domestic corporation. a. May be claimed only as tax credit. b. May be claimed only as tax deduction. c. Do not qualify either as a tax credit or as a tax deduction. d. May be claimed either as a tax credit or as a tax deduction at the option of the income taxpayer. 18. Statement 1. Government agencies performing essential governmental functions are subject to tax unless expressly exempted. Statement 2. Government agencies performing propriety function are exempt from tax unless expressly taxed. a. Both statements are correct. b. Both statements are incorrect. c. Statement 1 is true while statement 2 is false. d. Statement 2 is true while statement 1 is false. 19. Statement 1. Tax avoidance or tax minimization is the use by the taxpayer of legally permissible methods in order to reduce tax liability. Statement 2. Tax evasion or tax dodging is the use by the taxpayer of illegal means to defeat or lessen the payment of tax. a. Both statements are correct. b. Both statements are incorrect. c. Statement 1 is correct while statement 2 is incorrect. d. Statement 2 is correct while statement 1 is incorrect. 20. Which one among the following items below is included in the gross state? a. Revocable transfer. b. Transfer with reservation of certain rights. c. Transfer under general power of appointment. d. Transfer in contemplation of death which is onerous. Downloaded by John Paul Magbitang (johnpaulmagbitang04@gmail.com) lOMoARcPSD|15752181
  • 33. 21. Which of the following statement is correct? a. The final tax on compensation of special kind of non-resident aliens is 25% of the gross income. b. Interest income from a foreign currency deposit unit in the Philippines of a non resident alien is not subject to final tax. c. Informer’s reward is subject to final tax of 10% based on the 10% of the value of tax assessed or P1, 000,000 whichever is higher. d. Prizes exceeding P10,000 derived by non-resident alien not engage in trade or business here in the Philippines is subject to a final tax of 20% 22. The power to decide disputed assignment, refunds of internal revenue taxes, fees or other charges, penalties imposed in relation thereof, or other matters arising other the tax code or other laws or portions thereof administered by the BIR is vested in the: a. Secretary of Finance b. Commissioner of Internal Revenue c. Court of Tax Appeals d. Regional Trial Court 23. The commissioner of the Internal Revenue (CIR) is prohibited by law to look into the bank accounts of taxpayer, except when: a. Taxpayer is accused of heinous crime b. Taxpayer did not invoke his right to privacy during the tax audit. c. Taxpayer applies for compromise on tax obligation on account of financial incapacity. d. The CIR has reason to believe that taxpayer has filed a false or fraudulent return. 24. A VAT-registered realty company sells real property in the course of its business. On April 30, 2016, it has sold a lot under the following items (VAT excluded). Selling Price P4,000,000 Down payment, 4/30/16 400,000 1st Installment, 4/30/17 600,000 2nd Installment, 4/30/18 1,000,000 3rd Installment, 4/30/19 1,000,000 4th Installment, 4/30/20 1,000,000 Interest and other charges of 10% On unpaid balance per installment The zonal value of the lot at the time of sale is P4, 800,000. The output VAT for the installment received on April 30, 2017, is: a. P43,200 b. P86,400 c. P115,200 d. P129,600 Downloaded by John Paul Magbitang (johnpaulmagbitang04@gmail.com) lOMoARcPSD|15752181
  • 34. Output Tax (4,800,000 x 12%) = 576,000 Principal 600,000 / 4,000,000 x 576,000 = 86,400 Interest 3,600, 000 x 10% x 12% = 43,200 Total 129,600 25. Counting No. 24, the output VAT on April 30, 2020, is: a. P132, 000 b. P144,000 c. P156,000 d. P228,000 4,800,000 x 12% = 576,000 out put tax Principal 1,000,000 / 4,000,000 x 576,000 = 144,000 Interest 1,000,000 x 10% x 12% = 12,000 Total 156,000 26. Assuming that the real property in No. 24 is the residential lot sold for cash of P1, 750,000 (VAT not separately blend in the sales document) on April 30, 2016. Zonal value of the lot at the time of sale is P1, 900,000. The output VAT on the sale is: a. P187,500 b. P210, 000 c. P228,000 d. None, as it is exempt the VAT. 27. The lease of real or personal property is subject to: i. 7% gross receipt tax (GRT) if the lessor is the bank. ii. 3% percentage tax or 12% value-added tax (VAT) if the lessor is not a bank. a. No to I and II b. Yes to I and II c. Yes to I only d. Yes to II only 28. Ayala Land, Inc. (ALI) bought a parcel of land in 2014 for P7 million as part of its inventory of real properties. In 2016, it sold the land for P12 million which was its zonal valuation. In the same year, it suffered a loss of P6 million for selling another parcel of land from its inventory. These were the only transactions ALI had in its real estate business. Which of the following is the applicable tax treatment? a. ALI shall be subject to a tax of 6% of P12 million. b. ALI’s gain of P5 million shall be subject to holding period. c. ALI could deduct its P6 million loss from its P5 million gain. d. ALI’s P6 million loss could not be deducted from its P5 million gain. 29. This is an inherent limitation on the power of taxation: a. The rule of taxation shall be uniform and equitable. b. No law impairing the obligations of contacts shall be enacted. Downloaded by John Paul Magbitang (johnpaulmagbitang04@gmail.com) lOMoARcPSD|15752181
  • 35. c. Tax laws can not apply to the property of foreign governments. d. Charitable institutions, churches, parsonages, convents and all lands, buildings and improvements, actually, directly, and exclusively used for religious, charitable or educational purposes shall be exempt for taxation. 30. X took a life insurance policy of P5 million where the monthly premium is P10, 000. The proceeds will be paid to X after 25 years to the X’s estate should X die before completing the equivalent of 25 years payment. If the X outlived the policy, which of the following is correct? a. The proceeds will be part of X’s gross estate. b. The proceeds will be part of X’s taxable income. c. The proceeds will be party taxable estate and partly exempt. d. The proceeds will be party taxable income and partly exempt. 31. Containing the preceding number, except that after paying the equivalent of ten years premium, X transferred the policy to Y for P1.5 million and Y continued paying the monthly premium as they mature. After 10 years, X died. Which of the following is correct? a. The proceeds will be part of X’s gross estate. b. The proceeds received by Y is part of his taxable income. c. The amount received by X from Y is part of X’s taxable income. d. The amount received by the X from Y and the proceeds received by Y are partly taxable income and partly exempt. 32. An owner of several warehouses for rent, which used to be VAT- exempt because its annual gross receipts never exceeded P1,919,500 decided to register under the VAT system on January 2, 2016. The following data were from the first quarter ending March 31, 2016: Rental from warehousing services, net of VAT P 672,000 Purchases of supplies, gross of VAT 224,000 Inventory of supplies, January 1, 2016 201,600 Actual VAT paid on the inventory of supplies, January 1, 2016 21,600 The value added tax payable for the quarter is: a. P23,520 b. P35,040 c. P41,088 d. P52,608 VAT on rental (672,000 x 12 %) 80,640 VAT on purchase of supplies (224,000 x 12/112) (24,000) Transitional Input Tax(higher) (21,600) VAT Payable 35,040 Downloaded by John Paul Magbitang (johnpaulmagbitang04@gmail.com) lOMoARcPSD|15752181
  • 36. 33. Assuming that one vacant warehouse in the preceding number was rented out for the whole month of April 2016 and received P107, 000 as rent, gross of VAT but net of the applicable creditable withholding income tax (expanded). The output VAT on the rental receipt is: a. P12,000 b. P12,240 c. P12,840 d. P13,440 107,000/107% = 100,000 x 12% = 12,000 Numbers 34 to 42 are based on the following information: Dina Cabangon, a citizen and resident of the Philippines, died on November 1, 2016. Her marriage was under the system of absolute community of property. The following properties and obligations were left: Property received by Dina as inheritance on February 1, 2015 (during the marriage) P2,000,000 Real property acquired through the labor of both Dina and her Husband during the marriage (family home) 4,000,000 Property owned by Dina before marriage 300,000 Property owned by Dina’s Husband before marriage 200,000 Funeral expenses 300,000 Unpaid mortgage on property inherited 200,000 Judicial expenses for the settlement of the estate 80,000 Unpaid obligations (excluding the unpaid mortgage) 40,000 The property received as inheritance was part of the gross estate of the prior decedent at a fair market value of P1,100,000. At the time of inheritance, it was mortgage for P300,000. Dina was able to pay P100,000 before she passed away. 34. The total community property is: a. P6,500,000 c. P4,000,000 b. P4,500,000 d. P2,300,000 35. The total exclusive property is: a. P4,200,000 c. P2,300,000 b. P2,500,000 d. P2,000,000 36. The total ordinary community deduction is: a. P200,000 c. P420,000 b. P320,000 d. P445,000 37. The total ordinary exclusive deduction (excluding vanishing deduction) is: a. P400,000 c. P200,000 b. P300,000 d. P100,000 Downloaded by John Paul Magbitang (johnpaulmagbitang04@gmail.com) lOMoARcPSD|15752181
  • 37. 38. The deduction for family home is: a. P0 c. P2,000,000 b. P1,000,000 d. P4,000,000 39. The amount of vanishing deduction is: a. P0 c. P736,000 b. P816,000 d. P656,000 40. The total special deduction is: a. P0 c.P2,000,000 b. P1,000,000 d.P4,000,000 41. The taxable net estate is: a. P1,154,000 c. P3,154,000 b. P2,154,000 d. P4,244,000 SOLUTION: Separate Common Total 2,000,000 4,000,000 300,000 200,000 Gross Estate 2,000,000 4,500,000 6,500,000 Funeral Expenses: Actual 300,000 Limit (6.5 x 5%) 325,000 Threshold 200,000 Whichever is lower (200,000) Judicial Expenses (80,000) Indebtedness (200,000) (40,000) Vanishing Deductions* (736,000) Net Estate After Ordinary Deductions 1,064,000 4,180,000 5,244,000 Family Home (1,000,000) Standard Deductions (1,000,000) Share of Surviving Spouse 1/2 x 4,180,000 (2,090,000) Taxable NE 1,154,000 *Vanishing Deduction: Lower FMV 1,100,000 Mortgage Assumed and Paid (100,000) Initial Basis 1,000,000 Pro Rated Deductions: 1,000,000/6,500,000 x 520,000 (80,000) Final Basis 920,000 Vanishing Rate _ 80%__ Vanishing Deductions 736,000 42. Going back to the original problem, except that the marriage of Dina Cabangon to her spouse was under the system of conjugal partnership of gains. The taxable net estate would be: a. P3,206,032 c. P1,206,032 b. P2,206,033 d. P1,154,000 Downloaded by John Paul Magbitang (johnpaulmagbitang04@gmail.com) lOMoARcPSD|15752181
  • 38. Separate Common Total 2,000,000 4,000,000 300,000 Gross Estate 2,300,000 4,000,000 6,300,000 Funeral Expenses: Actual 300,000 Limit (6.5M x 5%) 325,000 Threshold 200,000 Whichever is lower (200,000) Judicial Expenses (80,000) Indebtedness (200,000) (40,000) Vanishing Deductions** (733,968) Net Estate After Ordinary Deductions 1,366,032 3,680,000 5,046,032 Family Home (1,000,000) Standard Deductions (1,000,000) Share of Surviving Spouse 1/2 x 4,180,000 (1,840,000) Taxable NE 1,206,302 **Vanishing Deduction: Lower FMV 1,100,000 Mortgage Assumed and Paid (100,000) Initial Basis 1,000,000 Pro Rated Deductions: 1,000,000/6,300,000 x 520,000 (82,540) Final Basis 917,460 Vanishing Rate 80% Vanishing Deductions 733,968 43. In March 2016, Imelda, who is fond of jewelries, bough the following: diamond ring for P750, 000; bracelet for P250, 000; necklace for P500, 000; and a brooch for P500, 000. Imelda drives income from the exercise of her profession as a topnotch Interior Designer. In October 2016. Imelda sold her diamond ring, bracelet and necklace for only P1.25 million, incurring a loss of P250, 000. She used the P1.25 million to buy a solo diamond ring in November 2016 which she sold for P1.5 million in September 2017. Imelda had no other transaction on jewelry in 2017. Which among the following best describes the tax implications arising from the aforesaid transactions? a. Imelda may carry over and deduct her 2016 loss only from her 2017 gain. b. Imelda may deduct her 2016 loss from both her 2017 professional income. c. Imelda may not deduct her 2016 loss from both her 2017 professional income and her gain. d. Imelda may carry over and deduct her 2016 loss from her 2017 professional income as well as from her gain. Downloaded by John Paul Magbitang (johnpaulmagbitang04@gmail.com) lOMoARcPSD|15752181
  • 39. 44. Taxation could be exercise by the following except one. Which one? a. Judiciary b. Legislative c. Local government unit d. President of the Philippines, in certain cases. 45. Stages, aspects or processes in taxation. a. Levy of the tax b. Collection of the tax. c. Payment of the tax by the taxpayer. d. All of the above. 46. Statement 1. Onerous donations are subject to donor’s tax. Statement 2. Gratuitous donations are not subject to donor’s tax. a. Both statement s are true b. Both statements are false c. Only statement 1 is true but not statement 2. d. Only statement 2 is true but not statement 1. 47. Which of the following statements are correct? a. Gift-splitting is a form of tax dodging. b. The uncle who is the brother of the donor’s mother-in-law is a non-stranger to the donor for purposes of the donor’s tax. c. A gift made to a relative in January 2016 is to be added to the gift made to the same relative in December 2015 in determining the gift tax. d. Renunciation by an heir including the surviving spouse of his/her share in the hereditary estate left by the decedent is subject to donor’s tax if done in favor of identified heirs to the exclusion or disadvantage of the other co-heir/s in the hereditary estate. 48. Mistah, single and sales executive of a leading pharmaceutical firm (RiteMed), received in 2016 the following from his employer: Salary, net of P267,000 withholding tax P 683,000 Allowances and benefits received:  Rent paid by RiteMed on the house which Mistah occupies for residential purposes, net of 5% withholding 129,200  Entertainment allowance subject to liquidation (P75,000 was duly receipted in the name of RiteMed and used to entertain RiteMed’s customers and the balance of P25,000 was used to purchase a late model mobile phone for the personal use of Mistah) 100,000  Reimbursement of entertainment expenses paid by Mistah (P17,500 was used to entertain Mistah’s boyhood pals and the balance of P22,500 was used to promote RiteMed’s businesses.) 40,000 Downloaded by John Paul Magbitang (johnpaulmagbitang04@gmail.com) lOMoARcPSD|15752181
  • 40.  Fixed yearly allowance for entertainment 85,000 The fringe benefit tax is: a. P50,400 b. P52,000 c. P84,000 d. P92,000 Gross Rent on Housing 129,200/95% = 136,000 x 50% = 68,000 Expense Account (CP & Pal Exp) 25,000 + 17500 42,500 Total 110,500 110,500/68% 162,500 162,500 x 32% 52,000 49. Continuing number 48, the income tax payable by Mistah is: a. P13,200 old law b. P27,760 c. P29,200 d. P43,360 Compensation(683,000 + 267,000) 950,000 Fixed Year Allowance 85,000 Personal Exemptions (50,000) Taxable Income 985,000 Income Tax per Tabular (OLD LAW) up to 500,000 125,000 Excess (485,000 x 32%) 155,200 Withholding Tax (267,000) Income Tax Payable 13,200 Income Tax per Tabular (TRAIN LAW) up to 800,000 130,000 Excess (185,000 x 30%) 55,500 Withholding Tax (267,000) Income Tax Payable (Refund) (81,500) Numbers 50 to 56 are based on the following information: Domestic Export Sales to private entities P500,000 P500,000 Sales to the government 500,000 Sales of exempt goods 500,000 Input taxes passed on by VAT-registered suppliers on: Sales to private entities 30,000 20,000 Sales to the government 25,000 Sales of exempt goods 10,000 Purchase of depreciable capital goods not attributable to any specific activity (monthly amortization for 60 months) 120,000 Downloaded by John Paul Magbitang (johnpaulmagbitang04@gmail.com) lOMoARcPSD|15752181
  • 41. The sales to the government were subjected to the automatic deduction of the 1% creditable withholding tax (CWT) on its purchases from domestic suppliers. 50. The value-added tax payable on the domestic sales to private entities is: a. Zero b. (P10,000) c. P60,000 d. P70,000 51. The total input taxes attributable to zero-rated sales is: a. P20,000 b. P30,000 c. P50, 000 d. P60, 000 52. If the input taxes attributable to zero-rated are claimed as tax credit, the net input value-added tax refundable is: a. Zero b. P40,000 c. P50,000 d. P60,000 53. The actual input taxes attributable to the domestic sales to the government is: a. P25,000 b. P30,000 c. P55,000 d. P65,000 54. The value-added tax payable on the domestic sales to the government which was withheld as final withholding VAT is: a. P5,000 b. P25,000 c. P30,000 d. P35,000 55. The journal entry to take up the domestic sales to the government is: a. Cash/AR 560,000 Sales 560,000 b. Cash/AR 560,000 Sales 500,000 Output tax 60,000 c. Cash/AR 535,000 Final withholding VAT 25,000 Sales 500,000 Output tax 60,000 Downloaded by John Paul Magbitang (johnpaulmagbitang04@gmail.com) lOMoARcPSD|15752181
  • 42. d. Cash/AR 530,000 Final withholding VAT 25,000 Creditable withholding tax 5,000 Sales 500,000 Output tax 60,000 Private Entities VAT 0% VAT Government OT 500,000 x 12% 60,000 60,000 500,000 x 0% -0- IT (30,000) (20,000) 500,000 x 7% (35,000) 120,000 x 1 / 4 (30,000) (30,000) VAT Payable -0- Excess Input Tax (50,000) FINAL VAT Withheld 25,000 56. The journal entry to reflect the excess of actual input VAT-over the statutorily allowed input tax on the domestic sales to the government is: a. Output tax 60,000 Revenue and expense summary 20,000 Input tax 55,000 Final withholding VAT 25,000 b. Output tax 60,000 Input tax 15,000 Final withholding VAT 25,000 Revenue and expense summary 20,000 c. Output tax 60,000 Input tax 55,000 Revenue and expense summary 5,000 d. Output tax 60,000 Revenue and expense summary 20,000 Input tax 55,000 Cash/AP 25,000 57. Which one among the following statements is wrong? a. Private Banks may be authorized to collect internal revenue taxes. b. The Bureau of Customs (BOC) is also charged with the collection of internal revenue taxes. c. The local government unit’s (LGUs), such as cities, Municipalities and Provinces, from part of the national tax system. d. The bureau of Internal Revenue (BIR) is part of the administrative machinery for the assessment and collection of internal revenue taxes. 58. The prescriptive period for the issuance of a formal letter of demand and final assessment notice (FLD/FAN) may not ordinarily be stayed because of the lifeblood theory. There are certain instances, however, where the running of the prescriptive period Downloaded by John Paul Magbitang (johnpaulmagbitang04@gmail.com) lOMoARcPSD|15752181
  • 43. may be suspended. Which among the following instances is not among the recognized exceptions which suspend the prescriptive period within which to assess? a. If the taxpayer is out of the country. b. If the taxpayer changes his address, informing the commissioner of such change. c. Where the taxpayer request for and is granted a re investigation by the Commissioner. d. When the Commissioner of Internal Revenue (CIR) is prevented from making an assessment and within 60 days thereafter. 59. An examination of a calendar year corporate taxpayer’s records shows that it filed its final adjustment income tax return on February 15, 2016 for its 2015 income. It subsequently filed an amended income tax return March 21, 2016. Up to what date is the Bureau of Internal Revenue (BIR) within which to issue a formal letter of demand and final assessment notice (FLD/FAN)? a. December 31, 2018 c. March 21, 2019 b. February 15, 2019 d. April 15, 2019 60. A closely held corporation has initially offered its shares in the Philippine stock exchange (PSE). The following data pertain to the initial public offering (IPO): Number of shares sold in IPO 1,000,000 shares Total outstanding shares before the listing in the PSE 3,000,000 shares Gross value in money of the IPO P20,000,000 The percentage tax due is: a. P100,000 c. P400,000 b. P200,000 d. P800,000 Gross Value of IPO 20,000,000 OPT Rate 1M / 4M = 25% ( 4%, from 4%,2%, 1%) 4% Percentage Tax 800,000 61. Shares of stock in a domestic corporation held as investment when sold not through the local stock exchange shall be subject to: a. 3% OPT or 12% VAT based on gross income b. ½ of 1% based on gross selling price or gross value in money. c. 5% on first P100, 000 capital gain; 10% on excess of P100, 000. d. 4%;2%;1% based on gross selling price or gross value in money. 62. Malakas and Maganda were legally separated. They have six minor children, all qualified to be claimed as additional exemptions for income tax purposes. The court awarded custody of two of the children to Malakas and three to Maganda, with Malakas directed provide full financial support for them as well. The court awarded the sixth child to Malakas’ parents with Malakas also providing full financial support. Assuming that only Malakas is gainfully earning while Maganda is not, for how many children could Downloaded by John Paul Magbitang (johnpaulmagbitang04@gmail.com) lOMoARcPSD|15752181
  • 44. Malakas claim as additional exemptions when he files his annual income tax return? a. Two children c. Five Children b. Three children d. Six children 63. Statement 1. Health and/or hospitalization insurance premium is deductible from gross income by the spouse who claimed the additional exemptions in case of married income taxpayers. Statement 2. Health and/or hospitalization insurance premium paid by an individual income taxpayer is deductible from gross income for a minimum amount of P2, 400 provided the family’s gross income for the year does not exceed P250, 000. a. Both statements are true b. Both statements are false c. Statement 1 is true while statement 2 is false. d. Statement 2 is true while statement 1 is false. 64. The following fringe benefits were given by an employer to its employees for the quarter ending September 30, 2016: De minimis benefits (not exceeding the maximum) P 200,000 Reimbursed expense of rank and file employees 400,000 Housing benefits to managers and supervisors (Representing total rents) 680,000 The fringe benefit tax payable for the quarter is: a. P160, 000 c. P442, 353 b. P320, 000 d. P502, 353 Housing Benefits to Manager 680,000 x 50% = 340,000 / 68% x 32% = 160,000 65. Continuing number 64, how much would be the total deductions from gross income which may be claimed by the employer? a. P1,100,000 c. P1,440,000 b. P1,280,000 d. P1,600,000 De minimis benefits (not exceeding the maximum) P 200,000 Reimbursed expense of rank and file employees 400,000 Housing benefits to managers and supervisors (Representing total rents) 680,000 Fringe Benefit Tax Expense 160,000 Total Deductions from Gross Income 1,440,000 66. Going back to the number 64, the employer shall file a remittance return of the final tax on fringe benefit and pay the tax withheld within: a. 5 days from the close of each month. b. 10 days from the close of each month. c. 10 days from the close of each fiscal quarter. d. 10 days from the close of each calendar quarter. Downloaded by John Paul Magbitang (johnpaulmagbitang04@gmail.com) lOMoARcPSD|15752181
  • 45. 67. Kalansay, a native of Negros, died leaving a property acquired by purchase from Naty Gok who died 3 ½ years ago. The property is now a Kalansay’s gross estate. The estate’s vanishing deduction rate is: a. 0% b. 20% c. 40% d. 60% Property is not gratuitously acquired by the present Estate 68. A lessor of residential units has the following gross receipts for 2016: Monthly Number Rent per unit of units Total P 9,000 5 P 540,000 10,000 5 600,000 11,000 5 660,000 12,000 5 720,000 The output VAT is: a. P0 c. P270,000 b. P230,340 d. P302,400 Leasing on a residential units is VAT Exempt 69. Using the same facts in number 68, only that the lessor is a VAT- registered person, the OPT tax due is: a. P0 c. P67,500 b. P57,585 d. P75,600 OPT Exempt also 70. The deduction allowed for the payment of premium on hospitalization insurance during the taxable year by a resident citizen amounting to P3, 000 for the months of August to December is: a. P 3,000 b. P2, 400 c. P1, 200 d. P1, 000 200 x 5 months = 1,000 71. The following are among the constitutional limitation on the power of taxation. Which one is not? a. Due process clause b. Non-impairment clause c. Equal protection clause d. No imprisonment for non-payment of tax Downloaded by John Paul Magbitang (johnpaulmagbitang04@gmail.com) lOMoARcPSD|15752181
  • 46. 72. Pacmom filed her income tax from return for 2015 on May 16, 2016 and paid the tax of P50,000. Upon audit by the BIR an assessment notice was issued on April 30, 2019, requiring Pacmom to pay a deficiency tax of P 75,000 not later than July 30, 2019. Pacmom will: a. False prescription as defense b. Request for an extension of time to pay the deficiency income tax. c. Go to the court of tax appeals to appeal the assessment made by the BIR. d. Ignore the assessment as the date of collection is already way beyond three years, covering taxable year 2015. 73. Date assessment was received March 08, 2016 Date petition for reinvestigation was filed by the BIR March 18, 2016 Date of filing of documents to support the petition April 08, 2016 Date decision of denial of the petition was received April 28, 2016 The last day to appeal to the CTA is on: a. April 17, 2016 c. May 8, 2016 b. April 17, 2016 d. May 28, 2016 74. Using the same facts in number 73, only that instead of going to the CTA, a request for reconsideration was filed with the BIR on May 7, 2016. Date decision of denial of the request for reconsideration was received on June 2, 2016. The last day to appeal to the CTA is on: a. June 7, 2016 c. June 23, 2016 b. June 21, 2016 d. July 2, 2016 75. Continuing number 74 only that instead of receiving a decision denying a request for reconsideration, a revised assessment was received on June 2, 2016. The last day to appeal to the CTA is on: a. June 7, 2016 c. June 23, 2016 b. June 21, 2016 d. July 2, 2016 30 days from June 2, 2016 76. Statement 1. The Bureau of Internal Revenue (BIR) has as its powers and duties the assessment and collection of the national internal revenue taxes, fees, and changes and the enforcement of all fortitudes, penalties, and fines connected therewith. Statement 2. The Bureau of Customs (BOC) has its powers and duties the assessment and collection of awful revenues from imported and exported articles and all other dues, fees, charges, fines, and penalties accruing under the tariff and customs laws. a. Both statements are correct. b. Both statements are wrong. c. Statement 1 is correct but statement 2 is wrong. d. Statement 2 is correct but statement 1 is wrong. Downloaded by John Paul Magbitang (johnpaulmagbitang04@gmail.com) lOMoARcPSD|15752181
  • 47. 77. A customs duty that imposes both advance and specific customs duties on imported or exported articles is: a. Anti-dumping duty. b. Countervailing duty. c. Discriminating duty. d. Compound customs duty. 78. Consider the following statements: i. The imposition of customs duties also assists in economic development. ii. Customs duties are sometimes imposed to protect local consumers. iii. The purpose of regular customs duties is to raise revenues to meet the needs of government. iv. Compound customs duties are computed only on the basis of units of measure, such as weight, measurement, quantity, etc. a. All the above statement is correct. b. Only statements I, II and III are correct. c. Only statements I, II and IV are correct. d. Only statement I, III and IV are correct. Numbers 79 to 80 are based on the following information pertaining to taxable year 2016: Phil. USA Sales P500,000 P600,000 Sales discounts, Returns and allowances 20,000 50,000 Cost of sales 230,000 250,000 Rent expenses 50,000 80,000 Salaries and wages 50,000 60,000 Interest expenses 10,000 - Entertainment and presentation 10,000 - Contribution to charitable institution 10,000 - Interest on bank deposit 10,000 20,000 Royalty- musical compositions 20,000 20,000 Dividends from domestic corporation 10,000 - 79. The taxable income if the taxpayer is a domestic corporation (DC) and resident citizen (RC), under: Itemized Deductions Optional Deduction DC RC DC RC a. P320,000 P270,000 P330,000 P280,000 b. P328,150 P278,150 P354,000 P592,000 c. P280,000 P230,000 P378,000 P328,000 d. P288,150 P238,150 P330,000 P568,000 ITEMIZED DEDUCTIONS DC RC Passive Income 40,000 40,000 Sales 1,100,000 1,100,000 Downloaded by John Paul Magbitang (johnpaulmagbitang04@gmail.com) lOMoARcPSD|15752181
  • 48. Sales Disc, Ret & Allowances (70,000) (70,000)____ Net Sales 1,030,000 1,030,000 Cost of Good Sold (480,000) (480,000) Business Gross Income 550,000 550,000 Total Gross Income 590,000 590,000 Other Expenses (240,000) (240,000) Interest 10,000 - (10,000 x 33%) (6,700) (6,700) Entert & Repre (1,030,000 x .5%) (5,150) (5,150) Net Income Before CC 338,150 338,150 CC Actual < Limit (10,000) (10,000) Personal Exemption -0- (50,000) Taxable Income 328,000 278,150 OPTIONAL DEDUCTION DC RC Passive Income 40,000 40,000 Net Sales 1,030,000 Business Gross Income 550,000 Total Gross Income/ Gross Receipt & Net Sales 590,000 1,070,000 40% OSD (236,000) (428,000) Personal Exemption -0- (50,000) Taxable Income 354,000 592,000 80. The taxable income if the taxpayer is a resident foreign corporation (RFC) and non-resident citizen (NRC): RFC NRC a. P160,000 P270,000 b. P130,900 P 80,900 c. P133,855 P 80,900 d. P120,000 P 70,000 ITEMIZED DEDUCTIONS RFC NRC Passive Income Sales 500,000 500,000 Sales Disc, Ret & Allowances (20,000) (20,000) _ Net Sales 480,000 480,000 Cost of Good Sold (230,000) (230,000) Other Expenses (100,000) (100,000) Interest 10,000 - (10,000 x 33%) (6,700) (6,700) Entert & Repre (1,030,000 x .5%) (2,400) (2,400) Net Income Before CC 140,900 140,900 CC 140,900 x 5% (7,045) (10,000) Personal Exemption -0- (50,000) Taxable Income 133,855 80,900 Downloaded by John Paul Magbitang (johnpaulmagbitang04@gmail.com) lOMoARcPSD|15752181
  • 49. TAXATION QUIZZER PART 3 1. The following are common to the inherent power of taxation, power of eminent domain and police power, except for which of the following? a. They are necessary attributes to the sovereignty. b. They interfere with private rights and property. c. They affect all persons or the public. d. They are legislative in implementation. 2. In case of ambiguity, tax laws shall be interpreted: a. Strictly against the taxpayer. b. Liberally against the government. c. Liberally in favor of the taxpayer. d. Liberally in favor of the government. 3. Which of the following statements is not correct? a. Taxes may be imposed to raise revenue or to provide incentives or disincentives for certain activities within the state. b. The state can have the power of taxation even if the constitution does not expressly give it the power to tax. c. For the exercise of the power of taxation, the state can tax anything at anytime. d. The provisions of taxation in the Philippine Constitution are grants of power and not limitation on the taxing power. 4. Which of the following is not a direct tax? a. Immigration tax c. Income tax b. Transfer tax d. Contractor’s tax (Now VAT) 5. In which situation will a CPA’s signature be necessary? a. When the value of the gross estate is P2,000,000 and above. b. When the value of the gross estate exceeds P2,000,000. c. Regardless of the value where the gross estate consists or registered or registrable property. d. When the value of the gross estate exceeds P200,000, although exempt from transfer tax. 6. A seller of goods is non-VAT registered. His annual gross sales amount to P1,919,500. To what business tax is he liable? a. 3% tax on VAT-exempt persons c. 3% common carrier’s tax b. 12% value-added tax d. Not subject to any percentage tax 7. Abigail sold through the local stock exchange, 10,000 PLDT shares that she bought 2 years ago. Abigail sold the shares for P2 million and realized a net gain of P200,000. The transactions is a. Subject to regular income tax rates for individuals but only 50% shall be recognized because it is a long-term capital gain. b. Subject to capital gains tax amounting to P15,000. c. Subject to percentage tax amounting to P10,000. d. Subject to regular income tax rates for individual under Section 24 (A). Downloaded by John Paul Magbitang (johnpaulmagbitang04@gmail.com) lOMoARcPSD|15752181
  • 50. 8. Proceeds of life insurance to the extent of the amount receivable by the estate of the deceased, his executor or administrator under policies taken out by the dependent upon his own life shall be I. Part of the gross estate irrespective of whether or not the insured retained the power of revocation. II. Not part of the gross estate if the beneficiary is irrevocable. III. Part of the gross income if the designation of the beneficiary is revocable. IV. Not part of the gross income irrespective of whether or not the insured retained the power of revocation. a. I and II b. I and III c. I and IV d. only I 9. One of the following is not correct Deduction Maximum a. Funeral Expenses P200,000 b. Family home P2,000,000 c. Medical expenses P500,000 d. Standard Deduction P1,000,000 10. One of them is not considered non-resident citizen? a. A citizen of the Philippines who establishes to the satisfaction of the Commissioner the fact of his physical presence abroad with a definite intention to reside therein. b. A citizen of the Philippines who leaves the Philippines during the taxable year to reside abroad, either as an immigrant or for employment on permanent basis. c. A citizen of the Philippines who works and derives income from abroad and whose employment thereat requires him to be physically present abroad most of the time during the taxable year. d. A citizen of the Philippines who went on a business trip abroad and stayed therein most of the time during the year. 11. Z is a Filipino immigrant living in the United States for more than 10 years. He is retired and he came back to the Philippines as a balikbayan. Every time he comes to the Philippines, he stays here for about a month. He regularly receives a pension from his former employer in the United States, amounting to USD1,000 a month. While in the Philippines, with his pension pay from his former employer, he purchased 3 condominium units in Makati, which he is renting out for P15,000 a month each. Does the USD1,000 pension become taxable because he is now in the Philippines? a. Yes. Income received in the Philippines by non-resident citizens is taxable. b. Yes. Income received in the Philippines or abroad by non- resident citizens is taxable. Downloaded by John Paul Magbitang (johnpaulmagbitang04@gmail.com) lOMoARcPSD|15752181
  • 51. c. No. Income earned abroad by non-resident citizens is no longer taxable in the Philippines. d. No, the pension is exempt from taxation being one of the exclusions from gross income. 12. Which of the following income earners is required to file income tax return? a. Minimum wage earners b. Non-resident alien not engaged in business c. An individual with respect to pure compensation income deriving from such sources within the Philippines, the income tax on which has been correctly withheld and that an individual deriving compensation from one employer at any time during the taxable year d. General professional partnership 13. Which of the following fringe benefit is not subject to fringe benefit tax? a. a. Housing benefit c. De minimis benefit b. b. Expense account d. Vehicle benefit 14. In the case of a taxpayer, only the following percentages of the gain or loss recognized upon the sale or exchange of a capital asset shall be taken into account in computing net capital gain, net capital loss and net income. I. Statement 1 One hundred percent (100%) if the capital asset has been held for not more than twelve (12) months by a taxpayer. II. Statement 2 Fifty percent (50%) if the capital asset has been held for more than twelve (12) months by a taxpayer. a. True, true c. False, false b. True, false d. False, true 15. 15.A, worked for a manufacturing firm but due to business reverses, the firm offered a voluntary redundancy program in order to reduce overhead expenses. Under the program, an employee who offered to resign would be given separation pay equivalent to his 3 months basic salary for every year of service. A accepted the offer and received P800,000 as separation pay under the program. After all employees who accepted the offer were paid, the firm found its overhead still excessive. Hence, it adopted another program, where various unprofitable departments were closed. As a result, B was separated from the service. B also received P800,000 as separation pay. At the time of separation both A and B have rendered at least 10 years of service but A was 55 years old while B was only 45 years old. As a result, a. Both amounts are exempt from income tax b. Both amounts are subject to income tax c. Only Mr. A is subject to income tax d. Only Mr. B is subject to income tax Downloaded by John Paul Magbitang (johnpaulmagbitang04@gmail.com) lOMoARcPSD|15752181
  • 52. 16. The following shall not be subject to estate tax a. The merger of usufruct in the owner of naked title b. The transmission or delivery of inheritance or legacy by the fiduciary heir or legatee to fideicommissary c. The transmission from the first heir, legatee, or donee in favor of another beneficiary in accordance with the desire of the predecessor d. All bequest, devises, legacies or transfers in social welfare, cultural and charitable institutions 17. Which of the following transactions is subject to zero-rated value- added tax? a. Services rendered to persons engaged in international shipping or air transport operations b. Services rendered banks, non-bank financial intermediaries c. Generation, transmission and distribution of electricity d. Services rendered by professionals such as CPA, Physicians, and Lawyers. 18. Which of the following businesses is allowed presumptive input value- added tax? a. Manufacturer of canned goods b. Manufacturer of packed juices c. Manufacturer of packed noodles d. Manufacturer of dried fish 19. Which of the following shall be allowed as a deduction from the gross income? a. Depreciation for vehicle for land transport used for personal purposes the value of which does not exceed P2,400,000. b. Depreciation for vehicle for land transport used in the business the value of which exceeds P2,400,000. c. Depreciation for land vehicles the value of which exceeds P2,400,000 where the taxpayer’s main line of business is transport operations or lease of transportation equipment and the vehicles purchased are used in said operations. d. Depreciation for airplanes and/or aircraft the value of which exceeds P2,400,000 where the taxpayer’s main line of business is sale of goods or properties and the vehicles purchased are used in said operations. 20. A, at the time of retirement, had 1,000 pieces of merchandise which was deemed sold at a value of P20,000, with an output tax of P2,400. After retirement, A sold to B 500 pieces of these for P12,000. In the contract of sale or invoice, A stated sales invoice number wherein the output tax on deemed sale was imposed and the corresponding tax paid on the 500 pieces. He prepared the following invoice: I. Gross selling price P10,800 II. VAT previously paid on deemed sale 1,200 III. Total P12,000 Downloaded by John Paul Magbitang (johnpaulmagbitang04@gmail.com) lOMoARcPSD|15752181